[Ответить в тред] Ответить в тред

02/12/16 - Конкурс визуальных новелл доски /ruvn/
15/11/16 - **НОВЫЙ ФУНКЦИОНАЛ** - Стикеры
09/10/16 - Открыта доска /int/ - International, давайте расскажем о ней!



Новые доски: /2d/ - Аниме/Беседка • /wwe/ - WorldWide Wrestling Universe • /ch/ - Чатики и конфочки • /int/ - International • /ruvn/ - Российские визуальные новеллы • /math/ - Математика • Создай свою

[Назад][Обновить тред][Вниз][Каталог] [ Автообновление ] 508 | 28 | 89
Назад Вниз Каталог Обновить

Определение N Аноним 03/09/15 Чтв 09:46:09  295488  
(60Кб, 580x734)
Этим тредом начинается цепочка, посвященная определению N. То есть натуральных чисел.
Итак, почему же аксиомы Пеано не определяют N?
Аноним 03/09/15 Чтв 09:51:35  295490
Нумералы Черча еще не обсирали?
Аноним 03/09/15 Чтв 09:54:58  295492
>>295490
Нет. Что это?
Аноним 03/09/15 Чтв 09:57:54  295493
>>295488 (OP)

О, спасибо камрад.
Аноним 03/09/15 Чтв 10:00:05  295495
Камень преткновения обычно - та самая индукция. Я вбрасывал логическую/теоретико-множественную интерпретацию индукции(имхо хорошо проясняющую дело), поясняющую её суть но её выпилили почему-то.
Аноним 03/09/15 Чтв 10:00:30  295496
>>295492
Формализация натуральных чисел в рамках комбинаторной логики. Через базисные комбинаторы определяется объект (комбинатор) "0" и операция (ну, тоже комбинатор) "+1" (инкремент). На их основе всё остальное.
Аноним 03/09/15 Чтв 10:05:39  295497
>>295496

Комбинаторная логика это формальная система. ФС использует индукцию на каждом шагу.

>>295495

Вбрось ещё.
Аноним 03/09/15 Чтв 10:13:14  295500
>>295497
Ловите стенку. На dxdy вбрасывал - там сразу в карантин за неиспользование TeX(ну не знаю я его).

Насчёт индукции - она эквивалентна утверждению, что (счётное ;) множество истинных утверждений {An} получается из истинного утверждения-базы A1, порождающего правила (An=>An+1) и логической аксиомы(или модуса, доказываемого в разных подходах мат. логики - кому как нравится) Modus Ponens. На мой взгляд, это утверждение интуитивно понятно, и непосредственного отношения к натуральным числам не имеет. Числа нужны только для индексирования(определения порядка) множества. Но ведь индексировать можно любым линейно упорядоченным множеством. Тут прослеживается связь с множествами, порождающей процедурой задания множества и логикой.

Видимо, понятие множества строк конечной длины и операции подстановок-конкатенации эквивалентно понятию числа(попутно вспоминаем нормальные алгоритмы Маркова). Но возможно ли на языке НАМ, задать арифметические операции для любого(неограниченной разрядности) числа?

Представляется, что число можно определить и на языке формальных грамматик(в которых работает рекурсия, а не индукция), не совсем уверен насчёт определения арифметических операций. Допустим алфавит {1}, X+Y->X|Y(единичная система счисления). Как записать, что XY->X|X|X...|X Y раз?
Например, так? XY->X|X(Y-1) где Y-1 - отщепление символа от Y, X1->X? Но тогда конкатенации не хватит, нужно уметь отщеплять/стирать символы.

Понятие числа также включает в себя понятие "полностью упорядоченного счётного множества", с точностью до биекции. Верно ли обратное? Можно ли, например, с помощью нескольких аксиом на полностью упорядоченном множестве - задать функцию сложения, например?
Еще есть связь с композицией операций - композицию одинаковых операций можно задать числом, но это те же строки вида aaaaa... по сути. С рекурсией тоже связь очевидна, как уже подмечено, потому что тривиальная рекурсия без ветвлений - тот же линейный порядок.

Так что довольно простой вопрос "А что такое N?" может навести на незамысловатые, но занимательные размышления.
Аноним 03/09/15 Чтв 10:19:40  295503
>>295497
ITT ищется неформальное определение N?
Аноним 03/09/15 Чтв 10:20:58  295505
>>295503
Whatever. Любые продуктивные определения, критика имеющихся определений. Как я понЕл.
Аноним 03/09/15 Чтв 10:29:56  295506
>>295500

Рекурсия же это форма индукции. А у Маркова с первых же строк идет определение N как рядов палочек, т.е. по сути оно берётся как исходное понятие.
Аноним 03/09/15 Чтв 10:33:14  295508
>>295506
>Рекурсия же это форма индукции
Двачую.
Чего тут против индукции-то имеют?
слоупок.жпг
Аноним 03/09/15 Чтв 10:38:40  295509
>>295508

Ничего, но надо прояснить статус всего этого. Что мы признаём исходным не определяемым, а что определяем как производное понятие.
Аноним 03/09/15 Чтв 10:41:28  295510
>>295508
Возмущение на индукцию пошло вроде с одного недалекого сосачера, недовольного тем, что определение индукции завязано на определении числа. Т.е. получается логическая петля.
От себя добавлю - что мат. индукция вроде как аксиома, но относится к "логическим" инструментам вроде доказательства от противного, простейшим модусам применяющимся при доказательствах(например тот же Modus Ponens a, a=>b |- b, и много других) - которые в рамках мат. логики можно рассматривать как свойства булевых функций, либо доказывать в рамках подходов исчисления высказываний. Т.е. выходим на истоки метатеории - которые сразу же ассоциируются с теоремой Гёделя и тематикой ИИ, автоматического вывода теорем. Отсюда вопрос - можно построить внятную и удобную метатеорию, в которой N(а также некоторые другие "аксиомы") будет определяться легко и безболезненно.
Аноним 03/09/15 Чтв 10:47:14  295512
>>295510
>определение индукции завязано на определении числа
Хуета какая-то школьная.
В этом смысле та же комбинаторная логика на определении числа ну вообще никак не завязана (хотя определение комбинатора индуктивно, да).
Т.е. поддвачну индукцию как логический инструмент. "Определять" её через числа нахрен не надо.
Аноним 03/09/15 Чтв 10:48:21  295513
>>295510
Поясню - логическая петя возникает в аксиоматике Пеано(определяем числа через индукцию - которая в свою очередь требует определение числа).
Еще сопутствующий вопрос - как можно из разных определений чисел выудить вменяемую хай-лвл мат. аксиоматику(необходимую для матана и прочего - тысячи их) вроде этой
http://ru.math.wikia.com/wiki/%D0%90%D0%BA%D1%81%D0%B8%D0%BE%D0%BC%D0%B0%D1%82%D0%B8%D0%BA%D0%B0_%D0%B2%D0%B5%D1%89%D0%B5%D1%81%D1%82%D0%B2%D0%B5%D0%BD%D0%BD%D1%8B%D1%85_%D1%87%D0%B8%D1%81%D0%B5%D0%BB
Ну то есть, если мы докажем всё для натуральных чисел - дальше не надо, с натуральных чисел "подъем" уже проработан Дедекиндом и последователями.
Аноним 03/09/15 Чтв 10:49:01  295514
>>295510

> индукция вроде как аксиома, но относится к "логическим" инструментам вроде доказательства от противного, простейшим модусам применяющимся при доказательствах

Т.е. ты объявляешь N как неопределяемое понятие подтверждая тем самым точку зрения твоего "недалекого сосачера".
Аноним 03/09/15 Чтв 10:54:40  295515
>>295512

Это он переврал. Говорилось обратное, что N полностью завязано на индукцию. Если считать индукцию как неопределяемое понятие то по сути у нас тут же возникает натуральный ряд, так как N это и есть какой-то базовый объект плюс применение к нему этой индукции.

Аноним 03/09/15 Чтв 10:56:59  295516
>>295514
ИМХО. Любая хрень о которой идет речь в аксиоматике - это неопределяемые понятия, т.е. определения для которых давать не надо - но можно приводить объяснения и примеры из реального мира. Иначе - уходим в бесконечную цепь определений, в какой-то момент нужно остановиться. По аналогии с естественным языком - "базовый" словарный запас усваивается интуитивно.
Я же объявляя N неопределяемым понятием не отказываюсь от выбора аксиоматики - а наоборот, рад всяким конструктивным предложениям. Вопрос - какую аксиоматику считать удобной, не так важно. Главное - чтобы не было логических петель и порочных кругов. Ну и хорошо бы, чтоб можно было ввести операции на натуральных числах и доказать их свойства, выводя их из простых и понятных аксиом - либо вычислительных моделей этих операций.(в рамках "метатеории" - для кого-то это может быть перебором, конечно). Потому что просто определение чисел без арифметических операций смотрится "немного" кастрированно лол.
Аноним 03/09/15 Чтв 10:58:42  295517
>>295513
>>295514
Эй, говнари, вы вообще что называете "индукцией"? Чего-то мне подсказывает, что вы путаете два несколько разных значения у этого слова, а именно "математическую индукцию" с, скажем так, "логической индукцией". Есть ещё электротехническое значение, его вы почему отбрасываете?
Пиздануться от вас, школоматематиков, можно. С терминологией не определились, но тред запилили.
Аноним 03/09/15 Чтв 11:05:50  295519
>>295517
мы(то есть я - >>295500,
>>295510, >>295513, >>295516) оперируем в терминах математической, а не классической гуманитарной логики. Под индукцией я понимал именно мат. индукцию. Вообще индукция - это вывод от частного к общему, в отличие от дедукции. В рамках исчисления предикатов дедукция представляется как подстановка термов, например, а индукция - навешивание квантора всеобщности на предикат
.
Про электромагнитную индукцию тоже вкурсе.

ЗЫ. И вообще я не школоматематик, а скорее шизоматематик - если уж в такой манере генерировать мемосы.
Аноним 03/09/15 Чтв 11:14:11  295522
Окей.

1. I, K, S - "комбинаторы";
2. () - операция аппликации такая, что если А и В комбинаторы, то (АВ) - комбинатор;
3. других комбинаторов нет;
4. для лубых комбинаторов x, y, z: Ix = x; Kxy = x; Sxyz = xz(yz).

Никакой мат. индукции не используется. Никакой, не нужна она тут. Число логические "договоримся о такой вот пизде". Чисто математический стиль "не ебу, что это за хуйня такая - I, K, S, черные ящики какие-то, но ведут они себя вот так-то".

Далее введем удобные краткие обозначения для показавшихся нам интересными комбинаторов:
0 := KI
+1 := S(S(KS)K)
+ := {много букв}
:= S(KS)K
Чем интересные? Просто тем, что при их подстановке в школьные выражения для натуральных чисел, получим верные тождества. Радуемся и начинаем для краткости называть все комбинаторы, которые можно получить из "0" аппликацией "+1" - натуральными числами.

Затем можно давать определение математической индукции, пользуясь этими "натуральными числами".
Аноним 03/09/15 Чтв 11:18:56  295524
>>295522
вот это заебись. Видимо то, что надо для меня.
Сразу вспомнил о "великом комбинаторе" чёт.
А индукция в принципе все равно используется по всей математике - без неё никуда. Просто её суть очень близка сути натуральных чисел(аксиоматика Пеано это подтверждает), может даже это изоморфизм с первого взгляда. ;)
А на аксиоматике Пеано можно элементарно и с минимальными усилиями определить операции и все школьные формулы(для натуральных чисел) вывести?
Аноним 03/09/15 Чтв 11:19:00  295525
>>295516

> Любая хрень о которой идет речь в аксиоматике - это неопределяемые понятия, т.е. определения для которых давать не надо - но можно приводить объяснения и примеры из реального мира. Иначе - уходим в бесконечную цепь определений, в какой-то момент нужно остановиться

Тут есть один ключевой момент, который меняет всё. Это рассуждение без всяких оговорок принимается когда объектная система и метасистема не пересекаются, например формализация геометрии. В нашем же случае эти системы пересекаются, как раз по понятию индукции.

> Я же объявляя N неопределяемым понятием не отказываюсь от выбора аксиоматики

Ну это уже вопрос вторичный. Нам бы разобраться с первичными определениями.

> Главное - чтобы не было логических петель и порочных кругов.

Так ты их и получаешь, если не признаешь индукцию неопределяемым понятием.

>>295517

Очевидно разговор про мат индукцию в широком смысле.
Аноним 03/09/15 Чтв 11:21:10  295526
>>295522

> Никакой мат. индукции не используется.

Используется, только неявно (из рекурсии).

Кроме того ты привел свои определения как часть формальной системы, а если откроешь любую книжку по матлогике где развивается теория ФС, там индукция будет на каждом шагу.
Аноним 03/09/15 Чтв 11:23:02  295527
>>295525
Как я понимаю, в аксиоматике Пеано мат. индукция является аксиомой, а не неопределяемым понятием. Претензия - как нам провести индукцию, не умея в числа. Мне кажется это реально сделать если использовать понятие "линейного порядка", т.е. мы утверждения не будем нумеровать, а просто упорядочим в порядке "порождения" их мат. индукцией.
Аноним 03/09/15 Чтв 11:25:44  295528
>>295526
>Используется
Так ты что индукцией-то называешь, можешь сказать?
Математической индукции, той, которая даёт механизм доказательств теорем, которая по сути "правило вывода" в определении не используется. Хотя бы потому, что это определение, а не доказательство.
Аноним 03/09/15 Чтв 11:26:37  295529
>>295527
То есть претензия к аксиоматике Пеано сводится к такой - каким боком мы проводим мат. индукцию по числам, нумеруем утверждения - если еще не знаем что это такое. Иначе появляется логическая петля - индукция и числа как бы сами себе насасывают.
Аноним 03/09/15 Чтв 11:30:02  295531
>>295527

Аксиомой она является внутри своей формальной системы. Но на уровне метасистемы тоже во всю юзают индукцию.

>>295528

Если я ничего не путаю в самой общей форме это называют абстрацией потенциальной осуществимости, то самое "и т.д." которое постоянно юзают в мат. литературе.
Аноним 03/09/15 Чтв 11:31:30  295532
>>295529
Ой а иди-ка ты нахуй. Логическая петля у него. Речь про определения, а не про доказательства. Логические петли в доказательствах порочны, а в определениях они указывают лишь на взаимосвязь, т.е. два якобы связанных петлёй определения рассматривай как одно целое. Но это не случай индукции и натуральных чисел.
Аноним 03/09/15 Чтв 11:35:42  295533
>>295532
Как ты можешь нумеровать утверждения(чтобы использовать мат. индукцию) - не зная, что такое число? Что-то должно быть определено раньше, чем что-то другое. Если определишь раньше числа - аксиоматика Пеано летит к хуям т.к. тебе нужны числа(в индукции) чтобы построить числа. Если индукцию - то возвращаемся к вопросу аксиоматики натуральных чисел. Можно избежать петли если не юзать аксиоматику Пеано.

Вообще аксиоматика Пеано мне представляется самой интуитивно ясной и лаконичной - только напрягает самоотсос с индукцией - может это я зря на неё гоню, но пока интерпретация у меня такая.
Аноним 03/09/15 Чтв 11:36:33  295534
>>295532

Лол, далеко пойдешь с таким подходом. Предлагаю начать с взаимности множества и совокупности, лол.
Аноним 03/09/15 Чтв 11:42:31  295535
Мне видится выходом вот что. Заюзать связь индукции с теорией множеств. В теории множеств постулируется факт, что если задана порождающая процедура элементов - значит задано и само множество. Почему бы не исключить из индукции числа, задав порядок утверждений A, B, C... - это конечно немного долбоебизм, но нам надо решить эту маня-проблему. Числа пропали - остался только порядок утверждений и правило по которому из предыдущего утверждения следует очередное ну и база индукции A. Индукция выступает в роли порождающей процедуры для множества утверждений - проблема вроде решена. Это только потом мы для удобства начнем нумеровать утверждения - нам похер на самоотсос - ведь в принципе он "только в нашем маня-мирке обоснований и интерпретаций".
Аноним 03/09/15 Чтв 11:49:10  295536
И еще - тут мне кажется, близки логические парадоксы, основанные на логической петле. Стоит её разорвать хотя бы формально(разрубить гордиев узел) - и проблемы как не бывало.
Аноним 03/09/15 Чтв 11:49:13  295537
>>295535
>порождающая процедура элементов
... которая будет "индуктивна". Вон пример такой процедуры, в >>295522
Чисто алгоритмическое описание процедуры построения множества. Да, оно индуктивно - в логическом смысле. Местный хуй от него нос воротит, считая, что буквы "и т.д." можно писать только зная, что такое число. Иди опиши процедуру, порождающую множество без этих букв.
Аноним 03/09/15 Чтв 11:52:06  295538
>>295537
Двачую. Считаю, что сам принцип "индуктивности" не завязан на числа, он всего лишь очень близок к нему и ассоциируется с ними - как мне кажется, потому в математике мы привыкли всё и вся нумеровать. Так что тут включается человеческий фактор.
Аноним 03/09/15 Чтв 11:57:28  295539
По теме определения N: а почему бы не определить по-школьному(ну или по-программистски)? Задать строки из {0, 1}(мы мыслящие компьютеры, да), и алгоритмы операций +, -, /, * а также отношений =, <, >? Там будет "индуктивность" т.к. алгоритм должен работать для строк из 0 и 1 любой длины. Но нам на это похуй, т.к. собственно мат. индукции там не всплывает.
Аноним 03/09/15 Чтв 12:03:59  295540
А вот для доказательства свойств операций придется применить мат. индукцию. Но нам пожалуй придется иметь дело со строками разной длины но мы, не будем их нумеровать по длинам - просто упорядочим. Положим базу, Выведем - для любой строки верно -> для следующей более длинной тоже, а значит вообще для всего множества строк(разной длины) свойство верно. Числа в мат. индукции не юзали, просто лайфхак с упорядочиванием.
Аноним 03/09/15 Чтв 12:10:22  295541
Не знаю как вы, но кажется я пришел к НирванеОсознанию™ что же такое N. Без когнитиного диссонанса, внутренних конфликтов, самоотсосов, и прочей хуйни. Чему весьма рад, чего и вам желаю. Выкатился.
Аноним 03/09/15 Чтв 14:04:19  295564
Сколько постов про определение N сделано, а метаматематики тут так и не показали, где конкретно в индукции N и почему без N индукции быть не может. Руками машут "ну тут базовый класс плюс 1". Похоже, что они прочитали в умной книжке рукомахания какого-нибуь Гёделя или Пуанкаре и теперь повторяют как попугаи.
Аноним 03/09/15 Чтв 14:35:57  295570
>>295541

Зря радуешься, просто это признак того, что ты мыслишь поверхностно и не видишь глубинных связей. Это хуёвое свойство для математика.
Аноним 03/09/15 Чтв 14:38:53  295572
>>295564

> где конкретно в индукции N

Говорилось что N это по сути только индукция. Так как определение базового объекта тривиально.

> почему без N индукции быть не может

Это не утверждалось. Хотя по сути это верно. Так как если у тебя есть индукция, то взяв базовый элемент ты по индукции получишь N. Так что формально это истинное высказывание, лол.
Аноним 03/09/15 Чтв 14:48:29  295577
>>295488 (OP)
> почему же аксиомы Пеано не определяют N
А зачем его определять? Берутся аксиомы, берется произвольное N и с ним работают. Если тебе не нравится N которое сипользуют другие люди, можешь взять свое.
Аноним 03/09/15 Чтв 14:53:50  295579
>>295577

Где ты возьмешь произвольное N, если у тебя ни одного не определено???
Аноним 03/09/15 Чтв 14:57:19  295581
>>295579
Выдумаю.
Аноним 03/09/15 Чтв 16:38:45  295604
https://en.wikipedia.org/wiki/Peano_axioms
Советую начать с чтения этой статьи.
Аноним 03/09/15 Чтв 17:27:11  295624
>>295604
Проиграл сам с себя и осознал, каким же я был дауном. Аксиома индукции в аксиоматике Пеано не является логическим приемом мат. индукции, хотя и похожа на него.

Я подменял понятие аксиомы индукции в аксиоматике Пеано "методом доказательства по индукции".
Я считал, что использование понятия числа идет в аксиоматике Пеано, потому что мы нумеруем утверждения и ссылаемся на понятие числа при использовании конструкта "логическое утверждение, зависящее от n" при применении метода индукции(отсюда делал вывод о логической петле и самоотсосинге).

Математические аноны, не повторяйте моих ошибок.
Аноним 03/09/15 Чтв 17:52:02  295628
>>295624

О, ещё один прозрел.
Аноним 03/09/15 Чтв 21:09:00  295677
>>295624
Ты это серьезно? Устроил весь этот анальный цирк из-за того, что не прочитал википедию внимательно?
Аноним 03/09/15 Чтв 22:25:17  295683
>>295677
Да, серьезно. Устроив анальный цирк - узнал и уяснил для себя некоторые интересные и полезные вещи. Вообще имхо - метатеории и "проблемы обоснования" являются анальным цирком и ментальной мастурбацией по сути своей(как и огромный кусок математики). Просто кому-то она доставляет - мне, например. Просто "прочитать статью", как бы "понять" - не интегрировав полученные знания в общую систему понятий и представлений, смириться и не дискутировать - это не мой подход. Лучше задать "дурацкий" вопрос и разобраться, чем всю жизнь сидеть и неверно что-то понимать. И вообще - дурацких вопросов не бывает - имхо.

А теперь разбор полетов aftermath.
Ну не въехал сразу в статью, наложил маня-предубеждения(в том числе из рассуждений других сосачеров) и получил полную логическую кашу в голове. Кстати англоязычная статья намного доходчивее(что неудивительно). Главное - это то, что я всё-таки объективно лучше стал въезжать в тему - хотя бы на крупицу. И не буду больше никогда спрашивать разъяснить эту хуйню - я её понял раз и навсегда. искренне надеюсь
Аноним 04/09/15 Птн 00:42:27  295698
Множество натуральных чисел есть множество конечных кардиналов.
Аноним 04/09/15 Птн 04:13:24  295709
>>295698
Как ты определяешь кардинал?
Аноним 04/09/15 Птн 04:38:44  295711
(14Кб, 384x384)
>>295709
>определяешь кардинал?
Очевидно же.
Аноним 04/09/15 Птн 05:55:43  295717
(5Кб, 275x183)
> Этот придурок даже создал отдельный тред
Аноним 04/09/15 Птн 06:02:37  295718
>>295500
> TeX(ну не знаю я его).
Потому что ты дегенерат и твоё знание математики -- на уровне кухарки.

>>295513
> ru.math.wikia

Правильно, что тебя, шизика, пидорнули на dxdy
Аноним 04/09/15 Птн 06:17:11  295720
>>295711
Это интересный вопрос, между прочим. В ZF нельзя определить кардинал привычным образом как класс всех равномощных множеств - потому что новые множества можно образовывать только из уже построенных множеств, а множества всех множеств у нас нет.
Аноним 04/09/15 Птн 09:32:46  295737
>>295720
Очень просто.
0)Есть пустое множество - ноль.
1)Есть множество {0} - один.
2)Есть множество {0,{0}} - два.
3)Есть множество {0,{0},{0,{0}}} - три.
И т.д.
Аноним 04/09/15 Птн 09:43:37  295741
>>295737

>И т.д.

Tell me moar.
Аноним 04/09/15 Птн 09:44:02  295742
>>295737
Это первые несколько натуральных чисел по фон Нейману. Но это не определение кардинала. Куратовский не смог определить кардинал в аксиоматике Цермело-Френкеля, и ему пришлось ввести дополнительную аксиому о порядковых типах. В ней кардиналы (точнее, порядковые типы) вводились как некие новые сущности, онтологически не связанные со множествами.

Короче, что такое кардинал?
Аноним 04/09/15 Птн 10:01:34  295743
>>295742
Мы говорим, что конечное множество имеет кардинал А, если оно равномощно одному из множеств >>295737
Фактически, кардинал - это обобщение представления о колличестве. Говорят, что множества имеют один и тот же кардинал тогда и только тогда, когда они равномощны.
С порядковыми типами проследуйте к ординалам.
Аноним 04/09/15 Птн 10:08:15  295745
>>295743
То есть ты отрицаешь, что кардинал - это самостоятельный, существующий объект, и считаешь, что словосочетание "имеют один и тот же кардинал" является просто синонимом словосочетания "равномощны"? Но ведь это же скучно.
Аноним 04/09/15 Птн 10:11:57  295749
>>295745
>скучно.
--> /b
Аноним 04/09/15 Птн 10:14:01  295751
>>295745
Я говорю о том, что кардинальное число - это свойство множества. Разумеется, на определённом уровне абстракции можно мыслить его, как самостоятельный объект, но это не меняет хода рассуждений, и высказывания о кардинале как о свойстве переводимы в высказывания о нём как об объекте.
Аноним 04/09/15 Птн 10:20:51  295752
>>295749
Математика нужна постольку, поскольку приносит фан.

>>295751
На кардиналах, как известно, можно построить развитую арифметику со сложением, умножением, возведением в степень, извлечением корней и логарифмированием. Концептуально удобнее думать о них как об объектах, нежели как о свойствах. Но в ZFC, кажется, кардинал нельзя определить как объект.
Аноним 04/09/15 Птн 10:28:43  295754
>>295752
Да, нельзя. Но если нас интересуют только кардиналы множеств, то удобнее говорить о них, как о самостоятельных объектах, а не заводить шарманку о множествах каждый раз. >>295751
Аноним 04/09/15 Птн 10:39:30  295755
Пиздец, каникулы закончились, а долбоёбы с /sci/ так и не свалили. N определяют, кардиналы, блять, мат. треды в говно скатили, дауны.
Аноним 04/09/15 Птн 10:57:09  295761
>>295755
У нас тут серьёзный дискасс. Соси.
Аноним 04/09/15 Птн 11:48:05  295776
>>295683
Ты не задал дурацкий вопрос, ты именно что устроил анальный цирк: никто не может определить N, конструкция по сути определяется через себя, ко-ко-ко. Если б мог четко и недвусмысленно сформулировать свою претензию, тебе бы давно ответили, сколь дурацкой бы она ни была.
Аноним 04/09/15 Птн 13:48:52  295799
>>295718
>>295717
Ну да, ну да - по теме треда ты ничего не можешь сказать, а говном полить - пожалуйста. С dxdy меня не пидорнули - мне просто в лом было дооформлять - тем более ответа я мог ждать годами. TeX я бы заюзал если бы мне оно надо было.
Ты сам-то чьих будешь? Профессор кафедры?
И еще - тред создал не я - >>295500, а значит вопрос интересен
кому-то еще.

Подгоревшие уебки-снобы-знатоки - проходите мимо, не задерживайтесь. Для вас каноничный мат-тред есть. "Тупые, щизики, низзя такое спгашивать, вы ньичего не панимаити!"

>>295776
Я уже объяснил почему нельзя что-то определять через само себя - чисто логически
>Что-то должно быть определено раньше, чем что-то другое.
Т.е. говоря более формально если построить граф ссылок определений - в них не должно быть циклов(что я и имел в виду под логической петлей). Простейший пример: понятие X использует в определении Y. Понятие Y использует в определении X. Пытаясь пройтись по "графу определений" (т.е. попытавшись уяснить что же такое X, либо Y) мы итоге попадаем в "порочный круг". А вот если представить такую ситуацию: X в определении использует Y, а Y - неопределяемое понятие - никаких циклов не возникает.

В случае с аксиомой индукции - работает второй вариант, как я понял.
Аноним 04/09/15 Птн 16:43:25  295855
>>295624
В следующий раз читай внимательнее.
Аноним 09/09/15 Срд 15:22:20  296164
(20Кб, 400x300)
Бамп. Хочу свежих мыслей и бугурта по сабжу.
Аноним 10/09/15 Чтв 19:10:30  296487
>>296164
Свежих не будет - математическая логика уже пятьдесят лет как мертвая наука.
Аноним 10/09/15 Чтв 19:32:38  296496
>>296487
тот самый момент, когда я не в курсе событий :)
На проблемы обоснования/формального вывода/проверки доказательств всем насрать, или просто всё что могли уже сделали? (Полу-)Автоматическая проверка доказательств - это же вроде довольно актуально, учитывая какие объемы выдают Перельманы, Монтидзуки и прочие?
Аноним 10/09/15 Чтв 22:36:38  296586
>>296496
Проблемы обоснования математики не существует. Это миф, который держится на поддуве философов.
Аноним 11/09/15 Птн 04:16:17  296667
>>296586
На поддуве тех, кто не понимает в философии. Решения нет и не может быть, поэтому нет и проблемы
Аноним 11/09/15 Птн 07:39:22  296668
>>296586
Крепко сказано. Тем временем, теоремами существования, формализмами(во имя строгости и обоснованности) и прочей "обосновательной" лабудой полнятся остальные разделы - ещё физики на это внимание обратили. Взять тот же Бурбаки-клаб к примеру. Может дело в том, что лично тебе мат. логика не нравится?
Аноним 11/09/15 Птн 08:15:38  296671
>>296668
>Бурбаки-клаб
Они не занимались матлогикой. Всё, что они сделали на эту тему, - написали одну-единственную книжку с одной специфичной формальной теорией, которая была нужна им, чтобы утверждать, что математические объекты онтологически являются всего лишь наборами символов. Идея, что Бурбаки был хардкорным матлогиком, не соответствует действительности, их трактат - довольно удобный, коротко и ясно написанный учебник фундаментальной математики. Упоротый матлогический трактат - это, например, Principia Mathematica, но не книжка Бурбаки.

Теоремы существования и единственности вовсе не относятся к основаниям математики. Последнее революционное событие в области оснований - создание после войны Tarski–Grothendieck set theory (аксиомы как в ZFC, плюс аксиома Тарского: для каждого множества существует содержащий его универсум Гротендика). TG ввела в науку большие кардиналы, после чего основания благополучно издохли. Кто ими сейчас занимается, просто копошится в дерьме вековой давности.
Аноним 11/09/15 Птн 08:57:09  296683
>>296671
Я имел в виду то, что проблемы "обоснования" и "строгости" возникают вне мат. логики. Приложения и область развития для мат.логики я вижу в создании систем, помогающих проверить формальные математические доказательства.
Аноним 11/09/15 Птн 08:58:20  296684
>>296683
Добавлю - возможно там и пригодятся метатеории и логики 2-го уровня.
Аноним 11/09/15 Птн 09:05:47  296686
>>296683
Я искренне сомневаюсь, что в естественных науках в принципе возможна строгость в математическом смысле.

Физикам в своей деятельности постоянно приходится воображать творческие модели, каждую конкретную из которых чрезвычайно трудно описать формально. В каждой физической задачке физику приходится выдумывать некоторое, иногда довольно большое, количество допущений. В механике, например, очень часто физику приходится полагаться на соображения симметрии. Отказ от всего этого превращает физику в математику. Попытки приложить матлогику к физике происходят с тех самых пор, как Гильберт сформулировал свою шестую проблему, и ни одного, ни единственного успеха уровня хотя бы Бурбаки до сих пор нет.

В химии нет не то что формальных теорий, но даже мало формализованных аксиоматических теорий. Ни одного аксиоматического изложения химии в природе вообще не существует. Поэтому человеку, который захочет приложить метаматематику к химии, придётся с нуля создать новую науку, аксиоматическую химию. Вряд ли у кого-нибудь сейчас найдётся достаточно бабла, чтобы сделать это.
Аноним 11/09/15 Птн 09:28:31  296687
>>296686
Я имел в виду приложение именно к математике(к другим областям).
Я думаю, есть в мат. логике потенциал для роста - но чё-то заглохло(может немодно, видится неактуальным - ко многим другим областям то же применимо). В области ИИ применяли же машины формальных доказательств, нашли кучу неизвестных теорем(в геометрии) - может бесполезных, но тем не менее. Правда там всё шло на уровне предикатов, а для полезных выводов нужно спускаться на уровень термов - т.е. создавать новые предикаты на основе термов(уравнения/неравенства/выражения).
Аноним 11/09/15 Птн 09:37:19  296690
>>295488 (OP)
Мне кажется вся сложность, противоречивость определения натуральных чисел заключается вот в чем. Вся современная математика опирается на теорию множеств. Теория множеств оперирует такими понятиями, как континуум, бесконечно-делимые интервалы. Вещественные числа, которые принципиально до конца не определены (мощность множества вещественных чисел - континуум). Натуральные же числа, они опираются на понятие дискретности (неделимого интервала). И вот в этом и есть вся суть. Дискретность и континуум это противоположные, противоречивые понятия, не сочетаемые вместе. Нужна другая математика, опирающаяся не на теорию множеств, а на что то другое. Что-то такое, у чего в основе лежит понятие дискретности. Так мыслю. Но вообще я дилетант. Поправьте, если где обосрался.
Аноним 11/09/15 Птн 09:37:54  296691
>>296687
В математике упадок. Речь идёт не о том, чтобы получать новые знания, а о том, чтобы хотя бы частично сохранить уже полученные. Количество людей, слышавших название "гомологическая алгебра", сокращается год от года.
Аноним 11/09/15 Птн 09:46:53  296693
>>296691
Так там вроде завязка на то, что количество и сложность необходимых знаний, время/труд необходимые на их усвоение, чтобы работать в какой-то области - увеличиваются, и причём не хило. Тут уже проблема ограниченности возможностей человеческого интеллекта встаёт, имхо.
>>296690
На повестке дня проблема натуральных чисел - дискретной природы. Проблем построения вещественных чисел, перехода дискретность-континуум мы не касаемся.
Аноним 11/09/15 Птн 09:59:07  296696
>>296691
Это полный бред, это просто невозможно. Количество людей на планете год от года растет, вычислительные мощности тоже растут, средства анализа совершенствуются. Причем всё это растет нехуевыми темпами. При таких условиях упадо математики невозможен, сосни хуйца.
Аноним 11/09/15 Птн 10:00:43  296697
>>296693
>завязка
Непонятно в чём. Возможно, в том, что власть на матфаках захватили сракодрищенские выблядки, которые ненавидят математику.
Аноним 11/09/15 Птн 10:01:23  296698
>>296696
идёт речь о настоящей математике, а не о том, что ты в школе проходил - и что можно решить в любом CAS.
Аноним 11/09/15 Птн 10:03:13  296699
>>296696
Возможен и происходит. Такое уже было в истории.
Аноним 11/09/15 Птн 10:14:18  296701
>>296699
Ну давай расскажи мне, когда блядь еще в истории человечество, обладая суперкомпьютерами (с вычислительной мощностью в петафлопсы), был упадок математики?
Аноним 11/09/15 Птн 10:21:31  296702
Век информационных технологий, когда у любого пидорахи буквально под рукой википедия, где можно прочитать про гомологическую алгебру и сраные аксиомы Пеано. Охуеть от всего прочитанного, увлечься и серьезно заняться математикой. А у них блядь, упадок.
Аноним 11/09/15 Птн 10:29:16  296703
>>296701
Один раз наука умерла в Древней Греции. Несмотря на успехи древнегреческой цивилизации, её математика отчего-то угасла.
Другой раз математика исчезла в исламском мире. Исламские фанатики запретили математику.
Ещё один раз это случилось в Третьем рейхе, когда нацисты поубивали всех еврейских математиков и, между прочим, не дали Цузе изобрести компьютер.

Я не знаю, какая сила убивает математику сейчас. Но, несомненно, математика угасает. Суперкомпьютеры есть, но они математике побоку.

>>296702
В наши дни серьёзные занятия математикой - это не исследования в стратегически важных, фундаментальных областях, не изучение гомологической алгебры, а тупое взятие интегральчиков под картофанчик. Не только в России, везде.
Аноним 11/09/15 Птн 10:31:44  296704
>>296702
Математику убивает потреблядство - косячок, дотка и прон с понями. Как и всё толковые интеллектуальные занятия. От твоих суперкомпьютеров умных и заинтересованных людей не прибавляется.
Аноним 11/09/15 Птн 10:36:32  296707
>>296703
>Суперкомпьютеры есть, но они математике побоку.
Ну ты же не прав. Вот прямо совсем. Например поиск простых чисел осуществляется как раз на таких компьютерах.
Только не надо мне рассказывать, что простые числа, как тян, не нужны
Аноним 11/09/15 Птн 10:40:16  296708
>>296707
ты путаешь разные науки/области. Чистая математика, к примеру - не о вычислениях и поиске каких-то там чисел, решений уравнений, коллизий хэшей. Этим занимаются прикладники/программисты. Computational science как есть.

Она больше о разных занимательных теориях, объектах/определениях, теоремах, гипотезах, выводах из них.
Аноним 11/09/15 Птн 10:49:35  296710
>>296707
>поиск простых чисел
Напомнило интервью Садовничего.

"Два наших аспиранта мехмата, специалисты по криптографии, несколько месяцев назад установили мировой рекорд. Суть в том, что надо разложить большое число на сомножители двух простых. Они считали несколько месяцев и разложили число 2 в, э-э, ... 27-й степени, это что-то бесконечное такое, и тем самым установили новый мировой рекорд в криптографии. Это произвело колоссальное впечатление на все центры - аспиранты мехмата на нашем суперкомпьютере установили такой мировой рекорд."
Аноним 11/09/15 Птн 10:59:29  296714
>>296710
Главное - рекорд же. Курс на топ-10, все дела. Срочно задрачиваем на рейтинги!
Аноним 11/09/15 Птн 11:03:40  296716
>>296710
Предлагаю новый рекорд - самое большое натуральное число, записанное в памяти компьютера(ов).
Аноним 11/09/15 Птн 11:05:31  296717
>>296714
Скажи, сколько тебе понадобится времени, чтобы разложить 227 на простые множители на бумажке?
Аноним 11/09/15 Птн 11:30:26  296724
>>296708
Ты очень коряво излагаешь свои мысли, но я понял о чем ты говоришь. Давай я тебе помогу сформулировать, о чем же на самом деле эта чистая математика. И после этого тебе сразу станет очевидно, почему она находится в упадке.
Чистая математика - это творческий процесс построения новых абстрактных математических объектов, структур. А также о связи между этими объектами. Суть этого творческого процесса в постоянном углублении уровня абстракций. Вот пример, который я уже постил как то раз в матан-треде:
1 уровень - простейшая чувственная абстракция. Переход от «двух яблок», «двух камней» и т. д. к понятию числа 2. При этом манипулирование предметами заменяется на универсальные законы работы с числами (или с преобразованиями, или с чем-то еще). Человек отделился от животных, стал разумным именно когда сумел выйти на этот первый уровень абстрактного мышления.
2 уровень абстракции возникает, когда понимаешь, что правила обращения с числами 2, 3, 15 и т. д. по сути одинаковы. Все эти числа можно складывать, перемножать, для них работают переместительный, сочетательный и другие законы. Иными словами, все целые числа «играют по одним правилам». Поэтому часто полезно оперировать не с конкретными числами, а с новым математическим объектом — кольцом целых чисел. Аналогично, разные повороты предмета в пространстве являются элементами нового математического объекта — группы трехмерных вращений.
3 уровень — это когда исчезает «осязаемость» элементов групп, колец, полей. Тут уже рассматриваются не конкретные группы вращений или иных преобразований, а просто абстрактные группы — совокупности элементов со строго очерченными свойствами. Здесь на первый план выходит то, какова структура группы, а не то, из чего она «состоит». Свойства всевозможных непротиворечивых математических структур, безотносительно к тому, где именно эти структуры возникают, изучает абстрактная алгебра.
4 уровень - теория категорий. Изучаются уже не конкретные группы, а сеть математических взаимосвязей между разными группами. Аналогично, изучается сеть взаимосвязей между самыми разными типами пространств или между самыми разными кольцами.
И так далее до бесконечности. Можно углублять уровень абстрации до бесконечности. Улавливаешь теперь суть вот этой самой чистой математики, на которую вы все надрачиваете?
Так вот, с углублением уровня абстракции сужается область применения в реальной жизни. Отсюда то самое выражение "говно без задач". Вот с этим как раз и связан упадок чистой математики. Уровень абстракции уже настолько глубок и сложен для понимания, а применяемость неочевидна.
Аноним 11/09/15 Птн 11:34:30  296726
>>296724
>И после этого тебе сразу станет очевидно, почему она находится в упадке
Мне вот не стало. Почему?
Аноним 11/09/15 Птн 11:38:55  296729
>>296726
Не знаю, но могу предположить, что ты просто туповат для осознания и понимания глубоких абстракций. Не у каждого человека достаточно развит интеллект. Многим не дается даже 2ой уровень.
Аноним 11/09/15 Птн 13:57:58  296773
(25Кб, 400x400)
>>296729
Я конечно совершенно зелёный в этих вопросах - но разве алгебраическая геометрия - это вся математика на сегодняшний день?
Аноним 11/09/15 Птн 14:06:40  296774
>>296773
http://www.mathnet.ru/php/archive.phtml?jrnid=intf&wshow=contents&option_lang=rus&viewarchiveID=10 - вот краткий курс основ современной математики.
мимошёл
Аноним 11/09/15 Птн 14:21:49  296775
>>296774
>> том 83
Этапом матана - зла немеряно
Лежит на сердце тяжкий груз
Аноним 11/09/15 Птн 14:29:24  296779
>>296775
Ват?
Аноним 11/09/15 Птн 14:33:21  296780
>>296773
Извини, я наверное тоже туповат, но я не могу понять причем тут алгебраическая геометрия и вот этот пост анона >>296729 ? Поясни.
Аноним 11/09/15 Птн 14:37:22  296782
>>296775
Планировали 276, но СССР развалился. Поэтому только 80.
Аноним 11/09/15 Птн 16:03:17  296806
>>296780
Признаю, я наверное всё-таки туповат - но пост >>296724 вызвал у меня ассоциацию с апологетикой Ъ-математики Вербитом и путём Дао, которому нужно следовать, как я понял(возможно неверно, в силу скудоумия) что всё должно быть завязано на алг. геом. как "передний край" современной математики - остальное "нинужно" и не развивается.
Аноним 11/09/15 Птн 19:31:37  296870
>>296806
Алгебраическая геометрия - не передний край, а базовая грамотность. Передний край - это сложная надстройка над >>296774
Аноним 11/09/15 Птн 20:21:29  296887
Передний край и задний край. Хуй и жопа математики.
Вы что, совсем ебанутые?
Что мешает изучать математику, почему вы так озабочены краями? Вы невзъебать какие учёные, которые уже постигли всю известную математическую науку, и теперь пытаетесь определить, какая область важнее?
Вы что, совсем ебанутые?
Передний край и задний край. Хуй и жопа математики.
Аноним 11/09/15 Птн 20:38:42  296898
>>296887
Можно изучать что хочешь, конечно. Можно задачки рашать, интегральчики считать. Но только ты будешь математиком второй культуры, а первая культура это алгеом.
Аноним 11/09/15 Птн 21:22:05  296920
Беда в том, что эти самые фундаментальные понятия часто определяются всем скопом. Т.е. невозможно определить понятие без определения всего семейства этих понятий.
Возьмём например основания геометрии. Аксиомы. Эти аксиомы определяют понятия (точка, прямая, плоскость, лежать на, етц.) через их взаимоотношения.
Вполне вероятно, что то же самое возможно и с натуральными числами.
Аноним 11/09/15 Птн 21:36:07  296929
>>296702

> у любого пидорахи буквально под рукой википедия, где можно прочитать про сраные аксиомы Пеано

А толку? Приходишь потом с этими аксиомами в места скопления математиков (на сосач например), указываешь им на явные противоречия в этом "определении" а в ответ жалкое бубу да бебе, кококо это философия, нам на это поххх...
Аноним 12/09/15 Суб 07:20:20  297046
>>296929
Где противоречия?
Аноним 12/09/15 Суб 11:31:49  297059
>>297046

Смотри аксиому индукции.
Аноним 12/09/15 Суб 12:20:01  297065
Зачем в целом современным учёным из области естественных наук изучать математику, когда существуют автоматические способы (тот же Wolfram Alpha, Mathematica) без знаний описать любое явление и найти значение любого выражения?
Аноним 12/09/15 Суб 13:00:34  297075
>>297065
Как минимум, чтобы понимать, что считать, как считать, и почему не считается.
Вот тебе скрин. Ищем траекторию полета одного небесного тела вокруг другого, решая дифур. Задача, которую решили аналитически еще в средневековье, формулы можно найти в любом учебнике по механике. Численно - решается, все нормально, орбита - эллипс. Аналитически - остается невычисленным. В самом конце другая система дифуров, чтобы убедиться, что нами все было введено правильно. Она считается.
Вопрос: какого хуя?
Аноним 12/09/15 Суб 13:01:18  297077
(56Кб, 796x906)
>>297075
Аноним 12/09/15 Суб 13:03:18  297078
>>297065
Потому что на выходе имеем кашу из чисел и совершенное непонимание того, что мы насчитали(в статистике, в моделировании, к примеру). А вычислительные/решательные скиллы по трактатам 18-го века для практиков и вправду особо не нужны(всеми ненавистные интегральчики и.т.д).
>>297059
ты бы тред прочитал, траль, или пройдем еще раз по кругу?
Аноним 12/09/15 Суб 13:06:37  297082
>>297078

> ты бы тред прочитал, траль, или пройдем еще раз по кругу?

Вообще-то я тот самый человек, который поднимал эту тему здесь на сосаче.
Аноним 12/09/15 Суб 13:14:24  297083
>>297082
Пошел нахуй тогда.
Аноним 12/09/15 Суб 17:37:50  297190
>>297059
Она не использует понятие натурального числа.
Аноним 13/09/15 Вск 03:41:23  297316
>>297190
А нахуя вообще для определения натуральных чисел аксиома индукции? Первые четыре аксиомы уже определяют N, а пятая нужна не для N, а для утверждений с элементами N. Убрать ее из списка, и ОП-шизик наконец-то успокоится.
Аноним 13/09/15 Вск 11:30:49  297335
>>297190

Понятие натурального числа использует её.
Аноним 13/09/15 Вск 11:31:32  297336
>>297316

Ты совсем что ли дурак, без нее ты только один объект определишь и всё.
Аноним 13/09/15 Вск 11:51:49  297338
>>297335
И?
Аноним 13/09/15 Вск 13:25:05  297348
>>297336
Ложь. Существуют бесконечные множества, являющиеся моделями для аксиом Пеано 1-4. Аксиома индукции нужна, чтобы у натуральных чисел были все известные свойства, например сложение.
knuebok 13/09/15 Вск 15:12:07  297357

>>295515
>Это он переврал. Говорилось обратное, что N полностью завязано на индукцию. Если считать индукцию как неопределяемое понятие то по сути у нас тут же возникает натуральный ряд, так как N это и есть какой-то базовый объект плюс применение к нему этой индукции.
Вот ты приводил тогда пример, что, по твоему мнению, определение производной не содержит порочного круга, а определение N содержит. Но ведь определение производной существенно опирается на понятие вещественных чисел, для которых нужна какая-никакая континуальность, которую мы, по сути, постулируем (аксиома полноты). То есть мы опять же постулируем утверждение, которое почти что эквивалентно определению производной, почему же это у тебя не вызывает никакого диссонанса?
P.S. не ответил на твою последнюю телегу, так как появились дела, а сейчас искать её лень.
Аноним 13/09/15 Вск 16:12:37  297365
>>297348
Оба хуйню несете. Без аксиомы индукции не доказано, что для каждого нат. числа кроме 1 есть предыдущее, т.е. что нат. числа не могут состоять из некоторого количества непересекающихся "ветвей" x, S(x), S(S(x)), ... . Но вот почему именно это утверждение не внести в качестве аксиомы (что для любого нат. числа отличного от 1 существует нат. число y такое, что S(y) = x) - вот это непонятно.
knuebok 13/09/15 Вск 16:20:16  297368
> Но вот почему именно это утверждение не внести в качестве аксиомы (что для любого нат. числа отличного от 1 существует нат. число y такое, что S(y) = x) - вот это непонятно.
Возьмём N, а сразу после него Z. Ну или в твоих терминах "ветвь" N, и "ветвь" Z.
Аноним 13/09/15 Вск 16:25:29  297370
(648Кб, 3200x2400)
>>297368
Ну да, я уже понял. Хорошо. А что если сказать, что натуральные числа совпадают с последовательностью 1, S(1), S(S(1)), ... ? В такой последовательности нет повторений и она принадлежит нат. числам, а эта аксиома проводит обратное вложение.
knuebok 13/09/15 Вск 17:21:24  297388
И как ты это предлагаешь записать на языке предикатов первого порядка? Схема аксиом индукции, собственно, и есть попытка это записать.
Аноним 13/09/15 Вск 17:44:27  297397
>>297388
Аксиома индукции, все-таки, нечто большее. Там говорится обо всех верных утверждениях, использующих нат. числа. Это излишне для определения самих нат. чисел. Чем они такие особенные, что только с ними можно херачить индуктивный переход? А если не особенные, то почему это вогнали в определение, словно без него это уже будут не нат. числа? Индукция же по сути привязана к многократной композиции функции (S(x) в данном случае), а не к нат. числам как таковым.
Как записать? Ну например (словами опишу) N равно минимальному по вложению множеству, удовлетворяющему предыдущему набору аксиом, т.е. с единицей и несклеивающей функцией следования. Любое такое множество будет содержать последовательность, порожденную единицей, а меньше нее уже не будет удовлетворять аксиомам, соответственно, минимум единственен по всем цепочкам вложений и потому определен корректно.
Аноним 13/09/15 Вск 17:46:44  297398
>>297397
И да, минимум среди всех множеств, у которых один и тот же элемент выбран в качестве единицы, и одна и та же функция в качестве функции следования.
knuebok 13/09/15 Вск 17:57:11  297402

>Как записать? Ну например (словами опишу) N равно минимальному по вложению множеству, удовлетворяющему предыдущему набору аксиом, т.е. с единицей и несклеивающей функцией следования. Любое такое множество будет содержать последовательность, порожденную единицей, а меньше нее уже не будет удовлетворять аксиомам, соответственно, минимум единственен по всем цепочкам вложений и потому определен корректно.
Мне кажется ты мешаешь всё в одну кучу. Если у нас есть теория множеств с её мощной аксиоматикой, то множество натуральных чисел и без всяких аксиом Пеано не составляет определить никакого труда, например, как наименьший счётный ординал.
Если у нас теории множеств нету, и мы хотим определить "теорию натуральных чисел" саму по себе, то мы имеем право пользоваться только языком первого порядка, в которых никаких множеств и их пересечений и вложений нету.
knuebok 13/09/15 Вск 18:00:36  297404
>то мы имеем право пользоваться только языком первого порядка
Немного не так сказал, мы имеем право пользоваться стандартными средствами, использующимися при определении теории первого порядка: задать свои константы, функциональные символы, символы отношений, задать аксиомы или перечислимые схемы аксиом по оговоренным правилам, которые описаны в любом учебнике логики.
Аноним 13/09/15 Вск 18:02:37  297406
Аксимома индукции нужна потому что ее нельзя доказать (как теорему) опираясь на остальные аксиомы. Ну или придумай доказательство, и тогда ее можно будет пидорнуть из аксиом.
Аноним 13/09/15 Вск 18:10:33  297407
>>297404
Постой-постой. Принадлежность элемента множеству мы писать можем - мы это делаем в аксиомах. Определить подмножество с его помощью не составляет труда: это множество с таким свойством, что если элемент лежит в нем, то лежит и в основном. Таким же точно образом мы накладываем условия на N в аксиомах Пеано, я не вижу тут ничего сверх "дозволенного". Если множество является подмножеством другого, то говорим, что оно вложено. Можно рассмотреть предел по вложению ака минимум. Все, какие проблемы?
Мы уже юзаем теорию множеств здесь. А вот что можно из нее юзать, а что - нет - об этом никто ничего не сказал.
Аноним 13/09/15 Вск 18:13:59  297408
>>297406
Я не говорю о том, что и без нее хорошо, я говорю о том, что она утверждает больше, чем требуется для того, чтобы определить нат. числа.
Аноним 13/09/15 Вск 18:25:21  297411
>>297407
Даже вот смотри, без всяких пределов:
Пусть зафиксированы элемент 1 и функция следования S(x). Назовем множество псевдонатуральным, если оно удовлетворяет аксиомам до аксиом индукции. Псевдонатуральное множество назовем минимальным, если никакое его подмножество не является псевдонатуральным. Теперь множеством натуральных чисел называется минимальное псевдонатуральное множество. Корректность определения, т.е. единственность, я уже показывал.
knuebok 13/09/15 Вск 19:10:15  297422
>>297407
Не хочу быть многословным.
>Принадлежность элемента множеству мы писать можем - мы это делаем в аксиомах.
Не можем. Мы можем это делать в теории множеств, так как бинарный предикат "принадлежать" там часть сигнатуры.
Аноним 13/09/15 Вск 19:22:03  297427
>>297422
>1. 0 is a natural number
>6. For every natural number n, S(n) is a natural number
Давай, запиши мне эти две аксиомы без теории множеств средствами логики первого порядка.
Аноним 13/09/15 Вск 19:32:41  297429
>>297427
>средствами логики первого порядка
1. A
2. для всех x B(x)
A := {0 is a natural number}
B(x) := {S(x) is a natural number}
Аноним 13/09/15 Вск 19:43:52  297430
>>297427
Это не аксиомы, а само собой разумеющиеся вещи. Грубо говоря, в теории натуральных чисел ничего, кроме натуральных чисел нету. Арифметика Пеано изложена, например, тут http://www.staff.science.uu.nl/~ooste110/syllabi/peanomoeder.pdf без всяких множеств.
Аноним 13/09/15 Вск 19:44:54  297432
>>297430
>Это не аксиомы, а само собой разумеющиеся вещи.
sic!
knuebok 13/09/15 Вск 19:52:45  297435
>>297430
Вообще тут есть некоторая путаница, аксиомами Пеано называют более менее 3 вещи.
Первая: аксиоматическое определение некоторого класса объектов в теории множеств, как, например, определение "группы" или определение "кольца". В этом случае о подмножествах говорить можно и даже нужно и аксиомы вроде (0 \in N) непременно нужны.
Вторая: определение абстрактной "теории натуральных чисел" как формальной теории исчисления предикатов первого порядка с константными символами (0,1) с функциональными символами арности 2 (+,*) (иногда заменяется просто на инкремент) и с одним предикатным символом арности 2 (<) (иногда не упоминается) и схемой аксиом индукции. В этом случае говорить о каких либо множествах просто бессмысленно.
Третье: определение абстрактной "теории натуральных чисел" как формальной теории исчисления предикатов второго порядка с константым символом (0) и одним функциональным символом арности 1 (S), и аксиомой индукции (которая пробегает по всем свойствам или, что почти то же самое, по всем подмножествам). В этом случае о подмножествах говорить можно, но не в том же смысле, что в первом случае.
В общем, человеку не в теме это объяснить довольно сложно, учебники бы почитали, что ли >_<
knuebok 13/09/15 Вск 19:55:52  297437
>>297432
Ссылку то почитал, или тебе лишь бы "сыграть" на разности бытового и математического смысла слова "аксиома" (которое в математике нисколько не "утверждение принимаемое без доказательства", а, скорее, "часть определения"), чтобы показаться чуть умнее непонятно перед кем?
Аноним 13/09/15 Вск 20:26:56  297443
>>297437
Если ты про аксиомы как элементы proof system (а точнее элементы индуктивного множества), то беру своё ёрничание обратно. Не ожидал встретить здесь человека хорошо знакомого с мат. логикой.
Аноним 13/09/15 Вск 20:31:07  297445
>>297435
Я не понимаю, что за дичь ты мне пытаешься тут втереть, если мы начали писать 1 \in S, то с какой это стати мы не можем определить и писать A \subseteq B ?
Сам для начала разберись, какую из трех вещей ты имеешь в виду, говоря мне, что я не могу писать 1 \in N, когда на вики, например, они именно в такой форме и изложены, и мы обсуждаем вики в первую очередь.
Аноним 13/09/15 Вск 20:35:04  297446
>>297430
Арифметика Пеано - это здорово, но мы пытаемся определить натуральные числа. Разница в том, что нат. числа - это самостоятельный объект, а арифметика - это набор доказуемых утверждений над этим объектом в такой-то модели.
Аноним 13/09/15 Вск 20:56:23  297448
>>297429
Что такое is a natural number? Я тупой комп, я английский язык нипанимайт.
knuebok 13/09/15 Вск 20:57:34  297449
>>297445
>если мы начали писать 1 \in S, то с какой это стати мы не можем определить и писать A \subseteq B ?
Я уже описал выше (и я подобного не утверждал).
>и мы обсуждаем вики в первую очередь.
Даже не знаю как и прокомментировать.
Аноним 13/09/15 Вск 21:03:30  297452
>>297449
>Я уже описал выше (и я подобного не утверждал).
Ничего ты не описал, Джон Сноу. И ты это утверждал: >>297422

>Даже не знаю как и прокомментировать.
Для справки, наше обсуждение начинается отсюда
>>295604
>>296929
Если на вики плохие, негодные аксиомы Пеано - неси свои. Тот пдфник с арифметикой Пеано не предлагать, потому что >>297446 . Ну или если это оно и есть, то ВСЕ ЯСНО
Аноним 13/09/15 Вск 21:06:15  297453
>>297448
Лал. А какой язык ты панимайт?
Аноним 13/09/15 Вск 21:07:34  297454
>>297453
Язык предикатов.
Аноним 13/09/15 Вск 21:12:25  297459
(269Кб, 735x1024)
Внезапно задам содержательный вопрос по теме треда. А какие существуют определения N, альтернативные определению Пеано?
knuebok 13/09/15 Вск 21:13:40  297460
Я уже перестал понимать что ты от меня хочешь. Ясен пень, что мы определяем наутральный ряд (ровно как и другой математический объект) буквами по бумаге, иначе никак мы и не можем просто потому, что мы люди и живём вот в таком вот плохом мире.
С моделями ситуация чуть сложнее, и я бы не хотел её касаться с учётом того что тут, видимо, и синтаксическую логику (теорию формальных языков, теорию доказательств) мало кто понимает.
Ну вот допустим определил ты через свои подмножества натуральные числа, теперь вопрос, а как определить множества? С помощью аксиом ZFC? А это разве будет "самостоятельный объект", а не "набор доказуемых утверждений над этим объектом в такой-то модели"?
Аноним 13/09/15 Вск 21:17:54  297466
>>297460
Натуральные определим с использованием теории множеств. Можно и не подмножествами, а наименьшим ординалом, без разницы. Просто при таком подходе получается, что аксиомы Пеано нихрена не дают (по крайней мере в таком виде, как это описано в вики), ибо с используемым ими инструментарием (теорией множеств) можно сделать все лучше без всяких чудо-аксиом.
Аноним 13/09/15 Вск 21:22:05  297468
>>297466
Ты же начал говорить, что нам незачем привлекать теорию множеств. Однако выясняется, что без нее удается только описать арифметику над N, а не само N. С учетом того, что мое изначальное утверждение было о том, что аксиома индукции - оверкилл для определения N, очевидно, что арифметика Пеано в этом смысле еще больший оверкилл.
knuebok 13/09/15 Вск 21:36:42  297474
>>297466
Не знаю что хотели сказать на википедии и мне, если честно, лень вчитывать, но вот держу сейчас в руках Cori, Lascar "Mathematical Logic Part II A Course with Exercieses", и там те же аксиомы что и на вики в разделе "First-order theory of arithmeti", но без глупых "принадлежит N" после квантора, более того странно, что они это самое "принадлежит N" в аксиоме индукции на x не навесили. Так профит становится понятен: мы можем исследовать формальную систему натуральных чисел (безотносительно всяких множеств) средствами матлогики и получать содержательные результаты.
Аноним 13/09/15 Вск 21:51:46  297484
>>297474
Хорошо, здорово, они обошлись без отношения принадлежности, но они не обшлись без арифметики Пеано:
> For each formula φ(x,y1,...,yk) in the language of Peano arithmetic, the first-order induction axiom for φ is the sentence ...
которая и так содержит N в виде нумералов, исходя из твоего пдфника. Нет, так можно определить N, я не спорю, но разве это не оверкилл - привлекать утверждения о истинности каких-то хитровыдуманных утверждений, включающих в себя формулы в такой-то теории над таким-то языком, когда мы просто хотим услышать ответ на вопрос "Что такое натуральные числа?"
Аноним 13/09/15 Вск 21:57:11  297485
>>297454
>Язык предикатов.
Ок. A и B(x) в твоём алфавите, значит. Так что всё ты понимаешь.
Расшифровка высказываний выходит за рамки логик.
Хотя, конечно, я неверно написал.
1. A(0)
2. For every x: A(x) => A(S(x))
Все буквы знакомы? Все значки знакомы? Осваивайся.
Аноним 13/09/15 Вск 21:59:10  297486
>>297459
Ну, типа такого, например: >>295522
Аноним 13/09/15 Вск 22:04:23  297490
>>297486
Нет, я говорю именно об альтернативных. На тифаретнике когда-то где-то писали, что бывают аксиоматики N, которые метафизически не уступают аксиомам Пеано, но не эквивалентны им.
knuebok 13/09/15 Вск 22:07:11  297495
>которая и так содержит N в виде нумералов
Так имена переменных - и записи аксиом это не часть теории (а часть метатеории).
> Нет, так можно определить N, я не спорю, но разве это не оверкилл - привлекать утверждения о истинности каких-то хитровыдуманных утверждений, включающих в себя формулы в такой-то теории над таким-то языком, когда мы просто хотим услышать ответ на вопрос "Что такое натуральные числа?"

Так аксиомы Пеано формулировали не для ответа на этот вопрос, в частности их формулировали для того, чтобы показать, что арифметизация математики возможна и сформулировать теорему Гёделя.
Вообще я тоже проблемы не вижу, но мне кажется я понимаю в чём видит её ОП, однако лучше будет, если он сам опишет, чем ему не нравится определение как "наименьшее индуктивное множество" в ZFC.
Весь день на саентаче просидел, гспди >_<
Аноним 13/09/15 Вск 22:13:02  297501
>>297495
Ему аксиома индукции не нравится, и все, связанное с индукцией - это все, что я знаю. Я докапывался до избыточности сей хрени, а он - до того, что она сама по сути порождает натуральные числа, то есть они как бы определены через себя. Впрочем, я бы сказал, что набор остальных аксиом уже определяет цепочку 1, S(1), S(S(1)), ... , которая по сути описывается индуктивным правилом, так что хули тут блядь.
Аноним 13/09/15 Вск 22:15:25  297503
>>297501
>по сути описывается индуктивным правилом
... но не использует математическую индукцию. Вот и вся высосанная из пальца проблема треда: индукция математическая путается с построениями индуктивного характера в целом.
Аноним 13/09/15 Вск 22:19:10  297504
>>297503
Ну некоторая логика в этом прослеживается, ибо сколько раз мы применили индуктивное правило (чтобы понять, другой ли мы элемент получили, или какой-то из предыдущих) - и есть натуральное число.
Аноним 15/09/15 Втр 10:58:24  297753
>>297316
Индукция нужна, как я понимаю - чтобы ввести арифметику, отношения равенства/неравенства на N и доказывать факты об N. Без неё будет generic линейного упорядоченного множества(незацикленного) - много из него не выжмешь.
Аноним 17/09/15 Чтв 10:37:43  298242
>>297357

Потому что мы явно оговариваем эту аксиому, какие тут могут быть вопросы? Мы же не от хорошей жизни это делаем, а по необходимости.
Аноним 17/09/15 Чтв 10:39:46  298243
>>297365

Ты узко смотришь на аксиому нидукции как только на цепочку символов в формальном языке. Ты понимаешь, что без содержательной индукции ты даже употребить многоточие в своём посте не имеешь права???
Аноним 17/09/15 Чтв 10:46:20  298247
>>297443

Ты не можешь дат определение натерального числа, при этом посмел назвать себя спецом по матлогике?
Аноним 17/09/15 Чтв 10:46:25  298248
делать вам нечего
Аноним 17/09/15 Чтв 10:49:19  298250
>>297452

Аксиомы Пеано не являются определением натуральных чисел, они лишь их описывают. А вопрос был как их определить, как математическйи объект. Чего вообще к вики привязались, это кривые попытки перепечатать куски учебников, зачем вообще на нее ссылаться???
Аноним 17/09/15 Чтв 10:49:55  298251
>>297454

Плохой ответ. Язык предикатов тривиальным образом содержит N.
Аноним 17/09/15 Чтв 10:53:14  298253
>>297474

Ты где вторую часть достал? В сети только первая валяется.

Аноним 17/09/15 Чтв 10:55:33  298255
>>297484

Так проблема не в хитрожопости, а что по сути ЭТО ОПРЕДЕЛЕНИЕМ НЕ ЯВЛЯЕТСЯ. Как я уже говорил, с тем же успехом можно приянть определение множества как совокупности. Просто разные термины и всё.
Аноним 17/09/15 Чтв 10:57:25  298257
>>298250
Аксиомы Пеано являются определением натуральных чисел.
Аноним 17/09/15 Чтв 10:59:29  298260
>>298257

>>298255

Аноним 17/09/15 Чтв 11:03:27  298263
>>297495

ZFC это пример формальной системы, фс в рамках финитных средств метатеории уже содержит тривиальное определение N. А значит определение N через ZFC это не определение а кодирование. Определением в математическом смысле оно быть не может, так как иначе будет содержать замкнутый круг, как здесь >>298255
.
Аноним 17/09/15 Чтв 11:32:31  298267
>>298263
В таком случае ни одного математического определения не существует.
Аноним 17/09/15 Чтв 12:03:02  298281
Knuebook, ты кто вообще? Давай своё резюме, вк раз уж под ником пишешь.
Аноним 17/09/15 Чтв 12:03:37  298282
>>298267

Как насчёт определения производной функции???
Аноним 17/09/15 Чтв 13:21:08  298343
(66Кб, 760x670)
>>298281
Достопочтенный господин, как ваш вопрос соотносится с тематикой треда?
knuebok 17/09/15 Чтв 15:52:46  298415
>>298263
Мы как-будто бы в разных дискурсах разговариваем. Разумеется (хотя поспорить с этим, при желании, и можно), у всех есть некоторое априорное представление об идее натуральных чисел и идее количества чего-нибудь там и, видимо, эти априорные представления неразрывно связаны с идеей индукции.
Однако, математика на текущем уровне строгости формально представляет из себя синтаксическую игру: у нас есть база данных фактов, которые представляют из себя строки на некотором заранее оговоренном языке. Изначально в неё положено несколько таких строк (аксиом) и мы можем по зараннее договоренным правилам получать из одних строк в этой базе некоторые другие строки и снова ложить их в базу. Цель игры для математика: путём таких вот "ходов" в ней (взятия нескольких строк из базы, и по договоренным правилам получения из них некоторой новой строки, после чего, возвращение этой новой строки обратно в базу). В этом случае, "определение" понимается ни в каком-то философском смысле, а очень банальным образом, определение представляет из себя следующую процедуру: мы берём некоторый значок "N" и некоторую строку phi на нашем языке. Вводим значок "N" в синтаксис нашего языка и договариваемся со всеми "игроками" этой игры, что когда они будут видеть N они будут подставлять вместо него строку phi. В ПРИНЦИПЕ в этой игре можно было вообще обойтись без определений, но это сделано для удобства играющих. Разумеется, эта игра описана с изрядной долей упрощения: есть такие вещи, как схемы аксиом, определения со свободными переменными, которые плохо вписываются в эту концепцию, о точных правилах игры можно прочитать в хорошем учебнике мат. логики. И тут я могу ответить на несколько возможных вопросов:
1) В математике есть определение со значком "N", которое заменяется на строку phi. Имеет ли этот значок и эта строка какое-то отношение конкретно к той идее натуральных чисел, которую мыслю я? Ответ: ХУЙ ЕГО ЗНАЕТ ЧТО ТЫ МЫСЛИШЬ, но, вообще говоря, играть в эту игру, ровно как и в шахматы, например, могут и люди никогда не задумывавшиеся а концепции "количества", так как её правила жестко описаны. Например, это может (и успешно) делать компьютерный ИИ.
2) Постойте, но разве те строки, что мы записываем в базу данных, они уже не есть "натуральные числа", получающиеся просто путём забывания различия между символами? Ответ: может есть, а может и нет, играющих в эту игру это не ебёт, они просто играют в игру.
3) Но разве эта игра тогда имеет какое-то отношение к реальному миру, и к реальным идеям таких категорий как "натуральное число", "количество", "треугольник", "длина", ...? Может имеет, а может и не имеет, а может и шахматы имеют к ним большее отношение, хуй знает. Но практика показывает, что люди, долго играющие в эту игру, иногда помогают другим людям строить ракеты. А иногда и не помогают.
Конечно, картина намеренно передёргнута и упрощена, но я тебе хотел показать, что на современном уровне строгости никаких "порочных кругов" не может быть в принципе. Задавай ответы.
knuebok 17/09/15 Чтв 15:57:57  298418
>Цель игры для математика: путём таких вот "ходов" в ней (взятия нескольких строк из базы, и по договоренным правилам получения из них некоторой новой строки, после чего, возвращение этой новой строки обратно в базу)
получить из исходных строк какую-то свою строку, которую он заранее сам для себя прилумал.
(Лень было вычитывать пост, посему он немного несвязным получился).
Аноним 17/09/15 Чтв 16:25:44  298444
>>298415

> Однако, математика на текущем уровне строгости формально представляет из себя синтаксическую игру

Чтобы иметь возможность играть в эту игру (имплементировать правила) требуются исходные понятия включающие в себя натуральные числа (называй как угодно - индукция, абстракция потенциальной осуществимости - суть думаю понятна). Поэтому то что ты будешь формулировать в рамках любой такой ФС (например аксиомы Пеано) уже не может в содержательном смысле определять N, иначе имеем очевидный замкнутый круг.

> В этом случае, "определение" понимается ни в каком-то философском смысле, а очень банальным образом

Ну так ясен пень, если так узко понимать понятие определения, то всё сойдется как ты пишешь. Ну это как назвать белое черным, а черное белым. Я же про общематематическое (содержательное) понятие определения всегда говорил. Именно поэтому я понимаемое в твоем смысле определение внутри ФС называл кодированием, чтобы не было путаницы. И оно ни разу не философское. По-твоему в любом учебнике по матану философия изложена когда на обычном языке определения даются?

> на современном уровне строгости никаких "порочных кругов" не может быть в принципе. Задавай ответы.

Как это нет, если они явно прослеживаются??? Легко живешь. По твоему подходу можно просто взять исходное понятие совокупности предметов а потом сказать что множество это совокупность и на этом основание математики построена (типа совокупность это интуитивное философское понятие, а нам, матемтикам похх на философию).

knuebok 17/09/15 Чтв 16:56:07  298453
>Чтобы иметь возможность играть в эту игру (имплементировать правила) требуются исходные понятия включающие в себя натуральные числа (называй как угодно - индукция, абстракция потенциальной осуществимости - суть думаю понятна). Поэтому то что ты будешь формулировать в рамках любой такой ФС (например аксиомы Пеано) уже не может в содержательном смысле определять N, иначе имеем очевидный замкнутый круг.
Но играющих в эту игру это не волнует. Ты путаешь совершенно два разных вопроса, первый: можно ли мыслящее существо, не имеющее никакой априорной идеи о количестве и натуральных числах научить играть в эту игру, и второй: корректно ли игроки играют в эту игру (тобишь корректно ли даются там определения)? Первый вопрос игроков в эту игру (тобишь, математиков) не волнует, они просто играют в игру. На второй вопрос ответ: да, все определения абсолютно корректны в том смысле, в котором это понимают математики.

>Ну это как назвать белое черным, а черное белым. Я же про общематематическое (содержательное) понятие определения всегда говорил.
Ну так я вот тебя расстрою, общематематическое определение именно таким и является. Например посмотри в каком виде вводят натуральные числа в metamath proof explorer http://us.metamath.org/mpegif/df-om.html просто вводят значок в синтаксис и договариваются, что его можно заменить на такую-то строку.
>Именно поэтому я понимаемое в твоем смысле определение внутри ФС называл кодированием, чтобы не было путаницы. И оно ни разу не философское.
Оно как раз философское, когда ты говоришь "кодирование внтури ФС" ты уже подразумеваешь, что есть какой-то "реальный мир множеств", а строки из ФС всего лишь "кодируют" эти "реальные объекты". Это как раз филосфская концепция, которая носит название "ультраплатонизм", но, к счастью, математическая строгость достигла такого уровня, что какой бы концепции не придерживался конкретный математик, если он "играет по правилам", другой математик всегда сможет понять результаты первого.
> По-твоему в любом учебнике по матану философия изложена когда на обычном языке определения даются?
Мне казалось что такие вещи объяснять не нужно. Тогда просто процитирую Бурбаков:
"...Математик, желающий убедиться в полной правильности, или, как говорят, “строгости", доказательства или теории, отнюдь не прибегает к одной из тех полных формализаций, которыми мы сейчас располагаем, и даже большей частью не пользуется частичными и неполными формализациями, доставляемыми алгебраическим и другими подобными исчислениями. Обыкновенно он довольствуется тем, что приводит изложение к такому состоянию, когда его опыт и чутье математика говорят ему, что перевод на формализо ванный язык был бы теперь лишь упражнением в терпении. Если возникают сомнения, то, в конечном счете они отно сятся именно к возможности прийти без двусмысленности к такой формализации — употреблялось ли одно и то же слово в разных смыслах в зависимости от контекста, на рушались ли правила синтаксиса бессознательным употреблением способов рассуждения, не разрешаемых явно этими правилами, была ли, наконец, совершена фактическая ошибка. Текст редактируется, все больше и больше приближаясь к формализованному тексту, пока, по мнению специалистов, дальнейшее продолжение этой работы не станет излишним."
>Как это нет, если они явно прослеживаются??? Легко живешь.
Они прослеживаются только тобой, что конкретно тебя должно было бы навести на мысли, но, видимо, не навело.
>По твоему подходу можно просто взять исходное понятие совокупности предметов а потом сказать что множество это совокупность и на этом основание математики построена
По моему подходу как раз нельзя, потому что сказать что-то можно только только на формальном языке, а на формальном языке слов "совокупность предметов" или "множество" нету.
Аноним 18/09/15 Птн 06:56:18  298597
>>298444
По-моему, твоя проблема в том, что ты пытаешься "понять" базовые математические сущности. Множества, совокупности... да похуям, что это такое. В пизду их определения в бытовом смысле -- т.е. в смысле что это. Тебе тут обесняют, что математиков в ФС интересуют некие правила, т.е. поведение определяемых объектов. Именно так следует понимать определения -- в смысле как это себя ведёт. Как себя ведёт множество и элемент множества? Да просто одно содержит либо не содержит другое, и последнее содержится или не содержится в первом. Ебал я в рот "совокупности" и вообще из какого материала состоит одно и другое, насрать вообще ЧТО это, мы просто определили в каких взаимоотношениях эти два понятия друг с другом. Это и есть абстрактная наука. То же самое с натуральными числами и аксиомами Пеано. Они не должны описывать, не описывают и не пытаются описать, чем являются натуральные числа; они просто декларируют, как они себя ведут и в каких взаимоотношениях друг с другом находятся.
Аноним 18/09/15 Птн 10:13:44  298609
Мне лень читать все излияния здесь. Я правильно понимаю, что речь идёт о ссылающихся друг на друга высказываниях:
1. В аксиоматике Пеано, натуральное число определяется индуктивно.
2. В определении индукции используется понятие натурального числа.

?
Аноним 18/09/15 Птн 12:29:15  298619
>>298609
Вот ты задаешь вопрос в треде, но делаешь это с сажей, без уважения...
Ты правильно понял - но всё уже разрешилось само собой. Индукцию по Пеано не следует путать с мат. индукцией(методом доказательства - там нам нужны натуральные числа). Тогда никаких проблем нет.
Аноним 18/09/15 Птн 17:54:43  298682
>>298619
С сажей я писал, чтобы не бампать тред лишний раз, поскольку ничего нового в тред я не писал, а лишь уточнил. Ты мог мне тоже ответить с сажей.

Что такое "индукция по Пеано"?
Аноним 19/09/15 Суб 07:06:16  298831
>>298682
>не бампать тред лишний раз
ты думаешь кто-то будет вечно сидеть в этом треде, когда истинные джентельмены давно обо всём договорились и разрешили ОП-вопрос?
>>298682
Аксиома индукции в аксиоматике Пеано, которая и являлась камнем преткновения.
Аноним 22/09/15 Втр 11:54:32  299652
>>298831

> когда истинные джентельмены давно обо всём договорились и разрешили ОП-вопрос?

Примеры в студию: кто, где и как это всё разрешил???
Аноним 22/09/15 Втр 11:56:08  299653
>>298609

Нет. Речь о том, что аксиоматика не может являться определением натуральных чисел, поскольку даже ее формулировка как формальной системы не может обойтись без использования содержательной индукции.
Аноним 22/09/15 Втр 12:05:36  299655
>>298597

Ок, тогда обратимся с теми же вопросами к правилам. Я вот, например, не понимаю индуктивных определений используемых в них. Научи меня.

> они просто декларируют, как они себя ведут и в каких взаимоотношениях друг с другом находятся.

Только вот описания этих аксиом само целиком построено на индукции, а это замкнутый круг.
Аноним 22/09/15 Втр 12:21:11  299659
>>298453

> Но играющих в эту игру это не волнует.

Оно волнует того кто устанавливает правила, потому что играющие в эту игру (собственно зачем им дали поиграть) решают в том числе и задачу обоснования понятий общих для обеих сторон.

> Ты путаешь совершенно два разных вопроса, первый: можно ли мыслящее существо, не имеющее никакой априорной идеи о количестве и натуральных числах научить играть в эту игру, и второй: корректно ли игроки играют в эту игру (тобишь корректно ли даются там определения)? Первый вопрос игроков в эту игру (тобишь, математиков) не волнует, они просто играют в игру. На второй вопрос ответ: да, все определения абсолютно корректны в том смысле, в котором это понимают математики.

Я не путаю, а изначально их разделял (и даже употреблял разные термины для определённости). И вопрос ставил с точки зрения обоснования понятия натурального числа, а в такой постановке не обойтись без одновременного рассмотрения как мета так и объектной теории поскольку по этой части они пересекаются (в метаматематическом смысле).

С каких это пор математиков не волнует корректность вводимых ими определений??? Хуйли с того что какого нибудь дифурщика не волнует определение базовых понятий, я говорю о систематическом построении математики с самого начала, а оно должно начинаться с явной формулировки исходных понятий и допустимых способов рассуждения. А поскольку индукция уже необходима на уровне формулировки общих понятий ФС, то определение чего-либо внутри ФС необходимо является производным от понятия индукции, а значит не может его обосновывать.

Более того, твоя попытка уйти от вопроса легко ломается. Я могу сказать, ок, забудем вообще про конкретные примеры ФС а обратимся с тем же самым вопросом к метаязыку - каков статус индукции в нем, является ли она неопределяемым понятием, или может быть сведена к более простым?

> Ну так я вот тебя расстрою, общематематическое определение именно таким и является.

Доказательств ты никаких не привёл, так что я не сильно расстроился.

Если ты не заметил, практически во всех основных подходах к основаниям математики это понятие берётся как исходное потому что оно минимально необходимое для возможности оперировать хоть каким-то видом бесконечности. Разные филсофии расходятся в вопросах использования "больших бесконечностей", а "в малом" они все общее ядро используют.

> Например посмотри в каком виде вводят натуральные числа в metamath proof explorer http://us.metamath.org/mpegif/df-
om.html просто вводят значок в синтаксис и договариваются, что его можно заменить на такую-то строку.

Этот пример вообще не в тему - назначение этой проги сугубо утилитарное и с выяснением связей между базовыми понятиемя
никак не соприкасается. Да и не может этого делать в принципе так как является всего лишь конеченой реализацией ФС в компе.

> Оно как раз философское, когда ты говоришь "кодирование внтури ФС" ты уже подразумеваешь, что есть какой-то "реальный
мир множеств", а строки из ФС всего лишь "кодируют" эти "реальные объекты". Это как раз филосфская концепция, которая
носит название "ультраплатонизм", но, к счастью, математическая строгость достигла такого уровня, что какой бы концепции
не придерживался конкретный математик, если он "играет по правилам", другой математик всегда сможет понять результаты
первого.

С какого перепоя ты увидел в моих словах ультраплатонизм??? Там даже и намека на умеренный платонизм не было, более того,
я даже за рамки финитного подхода Гильберта не вылезал.
Вообще, это глупый спор о терминах, ты просто обозвал философией основы содержательной математики. В любом случае тебе
нужно содержательное понимание при использовании метаязыка (иначе как ты будешь развивать терию ФС?). Так что просто
отмахнуться от этих вопросов нельзя.

> Мне казалось что такие вещи объяснять не нужно. Тогда просто процитирую Бурбаков:

У бурбаки вообще нет изложения мат логики, так что их кукареканье тут неуместно.

Я пытался пояснить тебе что имею ввиду говоря "содержательный язык" поскольку ты постоянно делаешь вид что не понимаешь
этого (хотя это общепринятый термин, в литературе по матлогике на русском довольно часто используется).

Кстати приписать формалистский подход всем авторам учебников по матану это круто. Думаю сказав им это в лицо, ты бы по
загривку мог отхватить за такое.

> Они прослеживаются только тобой, что конкретно тебя должно было бы навести на мысли, но, видимо, не навело.

Я приводил цитаты как минимум двух математиков полностью подтверждающих мои слова. Да это и не важно, главное я выдвинул тезис который ни ты ни твои ссылки (я их прочитал таки все) так и не опровергли. Всё что ты предлагаешь, это "прочувствовать", но это как ты должен понимать с точки зреиния математического подхода просто смешно. Если бы я был не прав, ты бы просто привел цепочку своих рассуждений из которых всё это было бы очевидно, а перед этим сломал бы мою.

> По моему подходу как раз нельзя, потому что сказать что-то можно только только на формальном языке, а на формальном языке слов "совокупность предметов" или "множество" нету.

ОК, объясняю на пальцах. Ты писал, что пусть даже на языке метаматематики мы юзаем индукцию, как только ввели аналогичные понятия в ФС то нас то что происходит на мета-уровне не волнут и мы можем без оглядки юзать те же самые понятия внутри ФС. Тогда что нам мешает взять в качестве мета-математики противоречивую теорию множеств, или еще хуже сформулировать определение ФС как производное от множества всех множеств (которое заведомо противоречиво) и дальше приказать себе забыть этот факт сидя внутри ФС. Ну и какова будет в этом случае ценность всех дальнейших рассуждений, если они заведомо сделаны на противоречивом фундаменте???


Аноним 22/09/15 Втр 12:30:45  299660
>>299659
ебать дебил.
Аноним 22/09/15 Втр 13:03:32  299670
>>299652
Сколько можно воду в ступе толочь?
Прочитай тред. Если не можешь - держи самые важные по моему мнению посты.
>>295799
>>295624
>>295538
>>295535
Если совсем маня-мирок не выдерживает - используй другие аксиоматики/определения чисел.
knuebok 22/09/15 Втр 13:39:39  299675
>Оно волнует того кто устанавливает правила, потому что играющие в эту игру (собственно зачем им дали поиграть) решают в том числе и задачу обоснования понятий общих для обеих сторон.
Нет, не решают, эту задачу решают другие люди, иногда те же самые, но это не входит в часть их деятельности как математика. Эта задача - не математическая, никто не пишет статьи с названием "Обоснование натуральных чисел", а если и пишут, то в журналы типа annals of mathematics они не попадают.
>Я не путаю, а изначально их разделял (и даже употреблял разные термины для определённости). И вопрос ставил с точки зрения обоснования понятия натурального числа, а в такой постановке не обойтись без одновременного рассмотрения как мета так и объектной теории поскольку по этой части они пересекаются (в метаматематическом смысле).
Ну тут есть два выхода: либо мы договариваемся о формальном языке, в котором будем рассматривать метатеорию и теорию одновременно, либо мы "рассматриваем" метатеорию и теорию не с математической точки зрения, и тогда уже никаких апелляций к математической строгости или "математическому парадоксу" не надо тут.
>С каких это пор математиков не волнует корректность вводимых ими определений??? Хуйли с того что какого нибудь дифурщика не волнует определение базовых понятий, я говорю о систематическом построении математики с самого начала,
Волнует, но определение "наименьший предельный ординал" абсолютно корректно, с точки зрения математиков.
>а оно должно начинаться с явной формулировки исходных понятий и допустимых способов рассуждения.
Ты опять либо намеренно сделал вид что не понял, либо не понял. Математики, чтобы заниматься математикой на принятом уровне строгости должны договориться о правилах синтаксической игры. Они могут на этом этапе использовать хоть идею индукции, хоть ученье Гротендика, хоть тетатры Ходжа, хоть в баню вместе сходить и одну и ту же шлюху трахнуть, главное одно: чтобы каждый математик после обсуждения таких правил (абсолютно не важно в каком ключе это обсуждения у них проходило) мог проверить "ход" другого и убедиться, что он корректный, при этом два разных математика должны прийти к одному и тому же "результату проверки" одного и того же хода. И вот после этого уже начинается "игра" - настоящая математика.
>А поскольку индукция уже необходима на уровне формулировки общих понятий ФС, то определение чего-либо внутри ФС необходимо является производным от понятия индукции, а значит не может его обосновывать.
Нет, оно не является производным понятием в математическом смысле так как "формулировки общих понятий ФС" - не были частью "математической игры", она началась позже.
>Более того, твоя попытка уйти от вопроса легко ломается. Я могу сказать, ок, забудем вообще про конкретные примеры ФС а обратимся с тем же самым вопросом к метаязыку - каков статус индукции в нем, является ли она неопределяемым понятием, или может быть сведена к более простым?
Если ты хочешь говорить о метаязыке и метатеории в математическом смысле (ровно как и о механике катящегося колеса, площади прямоугольного поля, позитрон-позитронных пучках) - ты должен сделать в "математической игре" корректные ходы, а именно, определить метатеорию, метаязык и тогда уже, с точки зрения "математической игры" о них можно будет что-то сказать.
>Доказательств ты никаких не привёл, так что я не сильно расстроился.
Получите, распишитесь:
https://en.wikipedia.org/wiki/Extension_by_definitions
https://en.wikipedia.org/wiki/Conservative_extension
>Если ты не заметил, практически во всех основных подходах к основаниям математики это понятие берётся как исходное потому что оно минимально необходимое для возможности оперировать хоть каким-то видом бесконечности. Разные филсофии расходятся в вопросах использования "больших бесконечностей", а "в малом" они все общее ядро используют.
Заметил, и что?
>Этот пример вообще не в тему - назначение этой проги сугубо утилитарное и с выяснением связей между базовыми понятиемя никак не соприкасается. Да и не может этого делать в принципе так как является всего лишь конеченой реализацией ФС в компе.
Этот пример просто сверх в тему. Это "математическая игра" в её самом идеализированном смысле, на практике же математикам приходится прибегать к естественному языку, и лишь хрупкая вера в то, что этот естественный язык всегда может быть однозначно двумя специалистами приведён к "ходам" в "математической игре" и поддерживает уверенность в том, что большинство математических текстов - математически строгие (то есть являются "записями ходов" в "математической игре").
>С какого перепоя ты увидел в моих словах ультраплатонизм??? Там даже и намека на умеренный платонизм не было, более того, я даже за рамки финитного подхода Гильберта не вылезал. Вообще, это глупый спор о терминах, ты просто обозвал философией основы содержательной математики.
Ты просто отбеливаешь философию, называя её "основами математики". Но то что спор глупый, с этим я абсолютно согласен.
>В любом случае тебе нужно содержательное понимание при использовании метаязыка (иначе как ты будешь развивать терию ФС?).
Не нужно, ровно как мне не нужно содержательное понимание "понимания", которое предоставляет нейронауки, чтобы заниматься математикой.
>У бурбаки вообще нет изложения мат логики, так что их кукареканье тут неуместно.
Чтобы понимать такие вещи, не нужно иметь за плечами монографию по мат. логике.
>Я пытался пояснить тебе что имею ввиду говоря "содержательный язык" поскольку ты постоянно делаешь вид что не понимаешь этого (хотя это общепринятый термин, в литературе по матлогике на русском довольно часто используется).
Видимо ты имеешь в виду "естественный язык" (который я действительно встречал в литературе по мат.логике), но надо чётко понимать, что это не математический термин и не математический объект, а значит про него нельзя делать никаких математических утверждений.
>Кстати приписать формалистский подход всем авторам учебников по матану это круто. Думаю сказав им это в лицо, ты бы по загривку мог отхватить за такое.
Ничего не могу посоветовать кроме того, чтобы перечитать цитату из бурбаков.
>Я приводил цитаты как минимум двух математиков полностью подтверждающих мои слова. Да это и не важно, главное я выдвинул тезис который ни ты ни твои ссылки (я их прочитал таки все) так и не опровергли. Всё что ты предлагаешь, это "прочувствовать", но это как ты должен понимать с точки зреиния математического подхода просто смешно. Если бы я был не прав, ты бы просто привел цепочку своих рассуждений из которых всё это было бы очевидно, а перед этим сломал бы мою.
Не видел ни цитат, ни (математических) рассуждений. (Но приводить их лично мне не надо, по причинам, изложенным ниже).
>ОК, объясняю на пальцах. Ты писал, что пусть даже на языке метаматематики мы юзаем индукцию, как только ввели аналогичные понятия в ФС то нас то что происходит на мета-уровне не волнут и мы можем без оглядки юзать те же самые понятия внутри ФС. Тогда что нам мешает взять в качестве мета-математики противоречивую теорию множеств, или еще хуже сформулировать определение ФС как производное от множества всех множеств (которое заведомо противоречиво) и дальше приказать себе забыть этот факт сидя внутри ФС. Ну и какова будет в этом случае ценность всех дальнейших рассуждений, если они заведомо сделаны на противоречивом фундаменте???
Отвечу вопросом на вопрос. Какова будет ценность всех шахматных партий, если изложить правила шахмат, пользуясь средствами противоречивой теории множеств? Ответ: ровно такая же, как и была до этого.

Вероятно, это моё последнее подобное сообщение, ибо диалог пошёл по второму кругу, а продвижений никаких.

Аноним 22/09/15 Втр 19:48:47  299765
>>299655
>Только вот описания этих аксиом само целиком построено на индукции, а это замкнутый круг.

Сейчас я дам определение натуральных чисел.
Введу технические понятия: символ и строка символов.

Введу постулаты.
PA. С любыми двумя строками можно совершить операцию конкатенации, результатом которой будет строка.
PB. Всякие две строки либо равны, либо не равны.

Если символами a и b обозначены строки, то утверждение, что a и b равны, я буду записывать как a = b, и утверждение, что a и b не равны, буду записывать как a ≠ b.

Введу аксиомы равенства. Они таковы.
EA. Всякая строка равна самой себе.
EB. Если символами a и b обозначены строки и если a = b, то b = a.
EC. Если символами a, b, c обозначены строки и если a = b и b = c, то a = c.

Введу аксиомы символов. Они таковы.
CA. 0 является символом.
CB. ' является символом.
CC. Каждый символ является символом либо на основании п.1, либо на основании п.2.

Введу аксиомы строк. Они таковы.
SA. Символ является строкой.
SB. Результат конкатенации двух строк является строкой.
SC. Каждая строка является строкой либо на основании п.1, либо на основании п.2.

Теперь я опишу те из строк, которые я буду называть натуральными числами.
NA. 0 является натуральным числом.
NB. Если символом n обозначена строка, являющаяся натуральным числом, то результат конкатенации n и ' является натуральным числом.
NC. Всякая строка является натуральным числом либо на основании п.1, либо на основании п.2.

Теперь я имею понятие натурального числа. Поскольку натуральные числа являются строками, к ним можно применять постулаты и аксиомы строк.
На основании PA, CB и NB заключаю, что для любого натурального числа, обозначенного символом n, существует натуральное число, получающееся конкатенацией n и '
. Это натуральное число, связанное с n, я буду обозначать n'.

Введу наконец последние аксиомы.
ND. Если символами m и n обозначены натуральные числа, то из m' = n' вытекает m = n.
NE. Если символом n обозначено натуральное число, то n' ≠ 0.

Итак, я закончил. Этот набор определений и аксиом не использует ничего, кроме естественного языка. В нём нет никакой индукции и используются лишь два символа, 0 и '. "Либо ... , либо ..." здесь исключающее, то есть выполняется один и только один из вариантов. Выдвигайте конкретные претензии, если они у вас есть.
Аноним 22/09/15 Втр 19:49:26  299766
>>299765
>>299655
>Только вот описания этих аксиом само целиком построено на индукции, а это замкнутый круг.

Сейчас я дам определение натуральных чисел.
Введу технические понятия: символ и строка символов.

Введу постулаты.
PA. С любыми двумя строками можно совершить операцию конкатенации, результатом которой будет строка.
PB. Всякие две строки либо равны, либо не равны.

Если символами a и b обозначены строки, то утверждение, что a и b равны, я буду записывать как a = b, и утверждение, что a и b не равны, буду записывать как a ≠ b.

Введу аксиомы равенства. Они таковы.
EA. Всякая строка равна самой себе.
EB. Если символами a и b обозначены строки и если a = b, то b = a.
EC. Если символами a, b, c обозначены строки и если a = b и b = c, то a = c.

Введу аксиомы символов. Они таковы.
CA. 0 является символом.
CB. ' является символом.
CC. Каждый символ является символом либо на основании п.1, либо на основании п.2.

Введу аксиомы строк. Они таковы.
SA. Символ является строкой.
SB. Результат конкатенации двух строк является строкой.
SC. Каждая строка является строкой либо на основании п.1, либо на основании п.2.

Теперь я опишу те из строк, которые я буду называть натуральными числами.
NA. 0 является натуральным числом.
NB. Если символом n обозначена строка, являющаяся натуральным числом, то результат конкатенации n и ' является натуральным числом.
NC. Всякая строка является натуральным числом либо на основании п.1, либо на основании п.2.

Теперь я имею понятие натурального числа. Поскольку натуральные числа являются строками, к ним можно применять постулаты и аксиомы строк.
На основании PA, CB и NB заключаю, что для любого натурального числа, обозначенного символом n, существует натуральное число, получающееся конкатенацией n и '. Это натуральное число, связанное с n, я буду обозначать n'.

Введу наконец последние аксиомы.
ND. Если символами m и n обозначены натуральные числа, то из m' = n' вытекает m = n.
NE. Если символом n обозначено натуральное число, то n' ≠ 0.

Итак, я закончил. Этот набор определений и аксиом не использует ничего, кроме естественного языка. В нём нет никакой индукции и используются лишь два символа, 0 и '. "Либо ... , либо ..." здесь исключающее, то есть выполняется один и только один из вариантов.
Аноним 22/09/15 Втр 19:50:29  299768
>>299765
Разметка проебалась, см. >>299766
Аноним 22/09/15 Втр 19:54:29  299769
>>299765
Ты мечешь бисер перед свиньей. Она тебя просто не поймет - не дано. Но всё равно респект, за отчаянную попытку что-то объяснить клиническому долбоебу.
Аноним 22/09/15 Втр 20:01:30  299773
>>299769
Я для себя писал, чтобы в будущем копипастить тем, кто интересуется.
Аноним 22/09/15 Втр 20:11:07  299775
Тащемта

>Введу аксиомы строк. Они таковы.
>SA. Символ является строкой.
>SB. Результат конкатенации двух строк является строкой.
>SC. Каждая строка является строкой либо на основании п.1, >либо на основании п.2.

это уже индуктивная вполне себе конструкция.
Аноним 22/09/15 Втр 20:36:28  299777
>>299775
Предпочитаю называть такие конструкции рефлексивными, ввиду как-то не особо замечаемого сейчас сходства с понятием this в ООП. Для использования этой конкретной конструкции не требуются ни использование на каком-то этапе трансфинитной индукции, ни какие-либо результаты из теории натуральных чисел, ни даже определение N. Поэтому её использование не есть порочный круг.
Аноним 22/09/15 Втр 20:36:44  299778
Мне кажется стоит напомнить про ДОМИНО
knuebok 22/09/15 Втр 21:21:29  299794
>>299777
Ну очевидно, что натуральные числа получаются из строк тупо забыванием различия между символами, поэтому строки, в каком-то смысле, конструкты сложнее, чем натуральные числа и, собственно, это и бомбило того куна на протяжении всего треда, но называть это противоречием в математике как-то глупо.
Аноним 22/09/15 Втр 21:51:58  299799
>>299766

Поздравляю, ты записал аксиомы Пеано без аксиомы индукции предварих изложение отрывочными сведениями теории формальных грамматик.
Аноним 22/09/15 Втр 21:57:19  299802
>>299777

Поясни подробнее про this и ООP.
Аноним 22/09/15 Втр 21:59:45  299804
>>298251
Что означает фраза "язык содержит множество"?
Аноним 22/09/15 Втр 22:02:12  299806
>>299766
Все уже давным-давно раскидано
>>297411
Аноним 22/09/15 Втр 22:09:13  299808
>>299806

Он не доказал существование.
knuebok 22/09/15 Втр 22:12:42  299809
>>299808
Таки да, тот кун в неявном виде использует аксиому выделения (переская все множества) и аксиому бесконечности (предполагая, что хотя бы одно такое множество существует), что гораздо более мощные штуки, чем индукция.
Аноним 22/09/15 Втр 22:21:11  299810
>>299799
>ты записал аксиомы Пеано
Нет, я записал не аксиомы Пеано. Я записал аксиомы, указанные Клини. Математическая индукция оказывается в моих определениях теоремой. Справедливость теоремы о матиндукции гарантируется, опосредованно, аксиомой NC.
knuebok 22/09/15 Втр 22:24:39  299811
>>299810
Только твою аксиому NC в языке первого порядка просто так "в лоб" не запишешь, схема аксиом индукции, по сути, и есть один из способов (со своими недостатками) записать NC.
Аноним 22/09/15 Втр 22:32:04  299813
(18Кб, 455x700)
>>299811
Естественно, она метаматематическая. Даже метамета, если подумать. Я предполагал, что сначала была обезьяна, которая умела лишь мычать. Постепенно её язык усложнялся, и наконец она прочитала свою первую книгу по логике для бывших обезьян, например пикрелейтед, и более-менее разобралась в том, что в ней написано. Я писал пост, ориентируясь на такую обезьяну.
Аноним 22/09/15 Втр 23:00:12  299819
>>299811
Аксиоматика предикатов/высказываний тоже - логика второго порядка(как правило, в каждой аксиоме формально там надо ставить конструкцию "для любого предиката выполняется"). И что характерно - аксиомы предикатов могут быть записаны на языке предикатов. Но т.к. это "мета" - приходится повышать порядок.
>>299775
Двачую, это индукция(ну или линейная рекурсия, кому как угодно) - в чистом виде. Ничего плохого в этом не вижу.
>>299794
А вот тут ты натолкнул меня на мысль. Между натуральными числами и строками есть взаимооднозначное отношение. Очень широко используемое на практике. Подумай какое.
Ответ: позиционная система счисления с основанием, равным мощности алфавита строки.
>>299811
Имхо ничего плохого в "метааксиоме" 2 lvl нет. Попытки избежать этого - весьма кривоваты и вымучены.
Аноним 22/09/15 Втр 23:04:17  299820
>>299813
Самые лучшие учебники логики на русском - церковно-семинарские. Это может вызвать некий когнитивный диссонанс, но, если задуматься, то не должно.
Аноним 22/09/15 Втр 23:04:33  299821
>>299809
Я не используя ни то, ни другое, я в явном виде предъявляю подмножество, которое принадлежит любому множеству, и эта принадлежность вытекает из набора аксиом до аксиомы индукции. Из того, что оно принадлежит всем, следует, что оно минимально.
Аноним 22/09/15 Втр 23:19:50  299827
>>299820
Нет, эти учебники говно. Есть нормальный учебник: Гладкий - Введение в современную логику, а далее уже идут более хардкорные вещи.
knuebok 22/09/15 Втр 23:51:11  299837
>>299821
>Назовем множество псевдонатуральным, если оно удовлетворяет аксиомам до аксиом индукции. Псевдонатуральное множество назовем минимальным, если никакое его подмножество не является псевдонатуральным.
Аксиома выделения два раза.
Существовние хотя бы одного псевдонатурального множества следует из аксиомы бесконечности
Аноним 23/09/15 Срд 00:09:56  299844
>>299837
>следует из аксиомы бесконечности
А можно ли изобрести бесконечность, из которой не следовало бы существование натуральных чисел? То есть можно ли вложить в понятие бесконечности смысл, отличный от "содержит счётное подмножество"?
knuebok 23/09/15 Срд 00:12:48  299845
>Аксиоматика предикатов/высказываний тоже - логика второго порядка(как правило, в каждой аксиоме формально там надо ставить конструкцию "для любого предиката выполняется").
Lol. Нет конечно. Обычные схемы аксиом.
> И что характерно - аксиомы предикатов могут быть записаны на языке предикатов. Но т.к. это "мета" - приходится повышать порядок.
Не очень ясная фраза. "Логика первого порядка" вообще теорией не является, так как там как минимум, есть правила вывода (а не только аксиомы).
>Имхо ничего плохого в "метааксиоме" 2 lvl нет. Попытки избежать этого - весьма кривоваты и вымучены.
По мне так вот эта вся "метаметааксиоматика на естественном языке" получилась очень кривоватой и вымученной (не в обиду тому куну), а стандартный подход - строен и логичен.
>Ответ: позиционная система счисления с основанием, равным мощности алфавита строки.
Не понял, к чему это.
knuebok 23/09/15 Срд 00:26:53  299847
>>299844
Множество называется бесконечным по Дедекинду, если оно равномощно некоторому своему собственному подмножеству. Множество называется бесконечным, если оно не равномощно никакому из множеств вида {1,2,3,...,n}.
В ZF (без аксиомы выбора!) может существовать множество, которое бесконечно, но не бесконечно по Дедекинду, оно будет меньше, чем счётное, но больше чем любое из множеств вида {1,...,n}
knuebok 23/09/15 Срд 00:29:31  299848
>оно будет меньше, чем счётное
Обосрамс. Оно будет несравнимо со счётным.
Аноним 23/09/15 Срд 00:32:33  299849
>>299845
>Lol. Нет конечно. Обычные схемы аксиом.
Я имел в виду предикаты, введенные через алгебраический подход(основные тождества, через которые всё можно вывести), а не через подход исчисления высказываний.
http://logika-predikatov.ru/logik/ravnosilnyie-formulyi-logiki-predikatov.html - ставим квантор всеобщности на все предикаты любой формулы(он подразумевается т.к. это тождества верные для любых предикатов) - вот и логика 2-го порядка.
>Ответ: позиционная система счисления с основанием, равным мощности алфавита строки.
Число, записанное таким образом является строкой с алфавитом {0,1,2,...} - с точностью до выбора символов цифр и одновременно натуральным числом - записанным довольно экономично(порядок числа ~ количество цифр - экспоненциальная связь). В представлении "единичной системы счисления" (a - 1, aa-2, aaa-3) длина строки ~ числу, а не его порядку - линейная связь. Так что вавилоняне или кто там далеко не дураки были, и даже поумнее римлян с непозиционной системой.
Аноним 23/09/15 Срд 00:35:45  299850
>>299847
А разве бесконечность и бесконечность по Дедекинду не эквивалентны? Можно моар инфы про это множество? Существование такого множества доказано или просто не исключено?
knuebok 23/09/15 Срд 00:39:26  299851
>>299850
>А разве бесконечность и бесконечность по Дедекинду не эквивалентны?
Без аксиомы выбора - нет.
>Можно моар инфы про это множество?
Я не специалист, не математик, не синтетик. https://en.wikipedia.org/wiki/Dedekind-infinite_set можно начать отсюда.
>Существование такого множества доказано или просто не исключено?
Доказано, что существуют модели ZF в которых подобные множества существуют. Грубо говоря, если добавить как аксиому, что такие множества существуют, то противоречий не появится, если сама ZF была непротиворечивой.
Аноним 23/09/15 Срд 00:43:34  299852
>>299851
Спасибо, весьма интересно. То есть возможно, что у кого-нибудь получится открыть мир бесконечностей, совершенно не похожих на привычные.
knuebok 23/09/15 Срд 00:45:50  299854
>>299852
Без аксиомы выбора очень мало структуры, чтобы такую "теорию мощностей" контролировать, красивой теории кардиналов, где все они выстроены в линеечку не выйдет, собственно не очень понятно, что там вообще без дополнительных аксиом можно содержательного сказать. Хотя, быть может, структуру удастся наростить другими аксиомами, не приводящими к теореме Кантора-Бернштейна.
Аноним 23/09/15 Срд 00:46:23  299855
>>299852
Уже сколько лет пытаются, так что вероятность крайне мала. С континуум-гипотезой слава Б-гу разобрались.
Аноним 23/09/15 Срд 00:49:13  299856
>>299837
Так-так-так, я же сразу сказал, что я использую теорию множеств - у меня как минимум определения на этом завязаны. Весьма странно ею не пользоваться, если мы пытаемся определить множество (N). Аксиома выделения тут тоже будет использоваться, т.к. мы в любом случае описываем N набором свойств. До того, более это мощные штуки или менее, мне дела нет - мне нужно не пользоваться аксиомой индукции, тем более она вообще не из теории множеств, так что какого черта мы ее вообще пытаемся сравнивать с аксиомами теории множеств?
Что касается аксиомы бесконечности - без нее мы тоже не обойдемся, поскольку множество N бесконечно, а доказывать мы все планируем через теорию множеств, т.е. сводя к аксиомам.
Так в чем претензия?
Аноним 23/09/15 Срд 00:52:05  299857
>>299854
>красивой теории кардиналов, где все они выстроены в линеечку не выйдет
Замечу, что теория, где все кардиналы выстроены в линеечку, как-то очень уж подозрительно проста. Кто сказал, что кардиналы нельзя организовать как-нибудь иначе? Расположить их в вершинах какого-нибудь "графа", например.
knuebok 23/09/15 Срд 00:59:00  299858
>>299856
Ну, если человек "не верит" в аксиомы Пеано, то вряд ли ему будет легче от конструкции, основанной на ZFC. Собственно претензия тогда в том, что для кого вообще нужно твоё определение тогда? Я думал ты хотел продемонстрировать возможность определить нат. числа не используя индукцию как идею вообще. А в ZFC и так есть "наименьшее индуктивное множество".
>тем более она вообще не из теории множеств, так что какого черта мы ее вообще пытаемся сравнивать с аксиомами теории множеств?
Ну теория нат. чисел с аксиомами Пеано без проблем интерпретируются в теории множеств, так что сравнивать, в каком-то смысле, можно.
>>299855
Уже нечто похожее есть же кстати. В тех же New Foundations привычных кардиналов нету.
Аноним 23/09/15 Срд 01:19:50  299864
>>299856
В теории множеств есть "индуктивность". Например, формирование множества конечного/бесконечного "порождающей процедурой" по сути та же индуктивность. Или вот формула счётного объединения, "максимального(т.е. наибольшей мощности) множества" и так далее.
Аноним 23/09/15 Срд 01:24:13  299866
>>299858
Но ведь мне хватило наивной теории множеств. Да, она плоха, но извините - в математике вообще хуй сыщешь хотя бы что-то из базовых понятий, что можно определить строго-по-хардкору, и при этом не охренеть от того, что при этом получилось.
>Я думал ты хотел продемонстрировать возможность определить нат. числа не используя индукцию как идею вообще. А в ZFC и так есть "наименьшее индуктивное множество".
Нат. числа сами являются индукцией как идеей. Определить их, не используя индукцию ни в каком виде - значит описать индукцию через не индуктивные аксиомы. Это та еще задача, я полагаю, если индукцию народ предпочитает брать в качестве аксиомы.
Аноним 23/09/15 Срд 01:36:31  299868
>>299864
Но зато нет аксиомы индукции, которая что-то утверждает обо всех утверждениях такого-то вида, т.е. на самом деле является набором из бесконечного числа аксиом. Очень плохая херь как аксиома. Формирование множества порождающей процедурой куда как проще и чище в этом плане.
А вообще - чего ты хочешь, если натуральные числа сами по себе являются структурой, описываемой индуктивным правилом? "Индуктивность" у них в крови. То, что ты (или не ты) там со строками накатал - в этом плане те же яйца, только в профиль, ибо вот оно, индуктивное правило:
>Если символом n обозначена строка, являющаяся натуральным числом, то результат конкатенации n и ' является натуральным числом.
Избавиться здесь от индуктивности, т.е. от утверждений, описывающих "следующие" элементы множества через "предыдущие", тебе не удастся, поскольку это будет то же самое, что доказать аксиому индукции через другие аксиомы. То есть лучшее, что ты можешь сделать - напридумывать несколько "менее индуктивных" аксиом, вывести из них аксиому индукции, и переписать определение N через них. Ну что ж, удачи с этим.
knuebok 23/09/15 Срд 01:42:12  299869
Я - ничего не хочу, со строками писал не я, но я и не утверждал нечто типа:
>Индукция же по сути привязана к многократной композиции функции (S(x) в данном случае), а не к нат. числам как таковым.
>Как записать? Ну например (словами опишу) N равно минимальному по вложению множеству, удовлетворяющему предыдущему набору аксиом, т.е. с единицей и несклеивающей функцией следования. Любое такое множество будет содержать последовательность, порожденную единицей, а меньше нее уже не будет удовлетворять аксиомам, соответственно, минимум единственен по всем цепочкам вложений и потому определен корректно.
теперь же ты пишешь
>А вообще - чего ты хочешь, если натуральные числа сами по себе являются структурой, описываемой индуктивным правилом? "Индуктивность" у них в крови.
То что тред повлиял на твоё "мировоззрение" если можно так выразится, это очень хорошо, но не надо вид делать, что "да очевидно же, что без индукции нельзя и я сразу это и имел в виду".
Аноним 23/09/15 Срд 01:42:24  299870
>>299868
Я не тот анон, который со строками мурыжился.
От себя скажу - за что вы так хейтите аксиому индукции? Подумаешь, что 2 лвл - у нас же метаматематика, наконец. Она же почти что - метод доказательства/логический приём(мат. индукция) - вы же его не хейтите - хотя его формулировка работает для любых утверждений(мета-лвл). Имхо, аксиома индукции вполне удобоварима и интуитивно ясна, как и доказательство по мат. индукции.
knuebok 23/09/15 Срд 01:44:30  299871
>От себя скажу - за что вы так хейтите аксиому индукции? Подумаешь, что 2 лвл - у нас же метаматематика, наконец. Она же почти что - метод доказательства/логический приём(мат. индукция) - вы же его не хейтите - хотя его формулировка работает для любых утверждений(мета-лвл). Имхо, аксиома индукции вполне удобоварима и интуитивно ясна, как и доказательство по мат. индукции.
Никто не хейтит, хейтит один нат.числа-кун, да и то не хейтит, а считает что это приводит к "порочным кругам" (в чём не прав нисколько), остальные же тут мимо проходили.
Аноним 23/09/15 Срд 02:24:05  299877
>>299868
>т.е. от утверждений, описывающих "следующие" элементы множества через "предыдущие"
A priori в утверждении "результат конкатенации n и ' является натуральным числом" не предполагается, что есть какие-то следующие элементы или предыдущие элементы. Здесь нет никаких утверждений о структуре строки, являющейся результатом конкатенации, и даже не говорится, что символ ' является её подстрокой, ибо понятие подстроки не введено. Здесь отнюдь не говорится, что каждое натуральное число - это ноль с некоторым количеством приписанных к нему штрихов. Доказательство существования порядка - нетривиальная задача. Оно существует, конечно, и его можно прочитать в книжке Ландау, но непосредственно в аксиомах наличие порядка не предполагается.
Аноним 26/09/15 Суб 14:53:13  300798
>>299827

Это тоже тупое говно. Вот у Манина пиздатые учебники.
Аноним 26/09/15 Суб 16:54:23  300843
>>300798
У манина как я знаю нет учебников по логике, у него есть только лекции по матлогике. Их гуманитарному гумусу не предложишь.
Аноним 28/09/15 Пнд 15:31:14  301190
>>299675

> Нет, не решают, эту задачу решают другие люди, иногда те же самые, но это не входит в часть их деятельности как математика.

Дело не в людях, я тебе про современное построение математики пишу, а ты всё про каких-то Васянов не желающих учить основы.
К тому же опять ты начинаешь убегать от ответа обозвав вопрос "не математическим". Точно так же говорили всякие мудаки (а среди них были и математики с мировым именем) в 19 веке, когда им в рыло предъявляли теоретико-множественные и логические парадоксы.

> Эта задача - не математическая, никто не пишет статьи с названием "Обоснование натуральных чисел", а если и пишут, то в журналы типа annals of mathematics они не попадают.

И что с того? Ты еще удивись почему они в журнал "Уравнения мат физики" не попадают.

> Ну тут есть два выхода: либо мы договариваемся о формальном языке, в котором будем рассматривать метатеорию и теорию одновременно, либо мы "рассматриваем" метатеорию и теорию не с математической точки зрения, и тогда уже никаких апелляций к математической строгости или "математическому парадоксу" не надо тут.

Ты сам попадаешь в ловушку своей же кривой логики в попытке уйти от ответа так как я элементарно могу задать тот же вопрос про метатеорию на естественном языке на которой ты собрался формализовать первую метатеорию и всё начнётся сначала.

> Волнует, но определение "наименьший предельный ординал" абсолютно корректно, с точки зрения математиков.

Для таких математиков и определение множества как совокупности объектов сойдет.

> Ты опять либо намеренно сделал вид что не понял, либо не понял. Математики, чтобы заниматься математикой на принятом уровне строгости должны договориться о правилах синтаксической игры. Они могут на этом этапе использовать хоть идею индукции, хоть ученье Гротендика, хоть тетатры Ходжа...

Ты утверждаешь что на уровне метаматематики естественного языка допустимы любые ничем не контролируемые методы. На это во-первых хотелось бы увидеть подтверждение в какой нибудь литературе или статьях (ссылки в студию). Во-вторых я уже вижу что это неправда, так как если ты возьмешь первый попавшийся текст обсуждающий начала любого подхода к основаниям математики, там первым делом ограничивают себя по максимуму в средствах вывода чтобы ненароком в говно парадоксов не наступить.

> главное одно: чтобы каждый математик после обсуждения таких правил мог проверить "ход" другого и убедиться, что он корректный, при этом два разных математика должны прийти к одному и тому же "результату проверки" одного и того же хода.

Это сверхсубъективный критерий, с позиций "математичности" о которой ты постоянно говоришь это намного хуже чем немного позаниматься "философией" исходных понятий (типа как я предлагаю).

> И вот после этого уже начинается "игра" - настоящая математика.

Ты фундаментально путаешь (или не до конца понял финитный метод Гильберта) творческую деятельность математика специалиста какого нибудь раздела математики с систематическим изложением этой науки с нуля. Я же нигде не отрицал, что на практике удобно вообще платонистский подход использовать (при этом держа в уме возможность формализации).

> Нет, оно не является производным понятием в математическом смысле так как "формулировки общих понятий ФС" - не были частью "математической игры", она началась позже.

Оно производное в том смысле, что без него ты не сможешь даже сформулировать и развить понятие ФС. А значит оно используется неявно.
Эта страусиная попытка закрыть глаза конечно забавная. Но ты должен быть последовательным, если ты не признаешь "нематематические" понятия за пределами ФС, какого хера ты их используешь в метаязыке? Выкручивайся без них тогда. Они же философские, а ты математик ёпта.

> Если ты хочешь говорить о метаязыке и метатеории в математическом смысле - ты должен сделать в "математической игре" корректные ходы, а именно, определить метатеорию, метаязык и тогда уже, с точки зрения "математической игры" о них можно будет что-то сказать.

Это будет еще один замкнутый круг который только добавит ненужное усложнение, так как наша метаматематика на естественном языке и так минимальна. Вводить ее формализацию для нашей задачи обоснования N нет, так как новая метаматематика всё равно будет такой же.

> Получите, распишитесь:

Зачем ты опять принес какие-то нерелевантные ссылки? К тому же это определения, а отвечал ты на реквест доказательств.

> Заметил, и что?

Охренеть, так с чем ты тогда споришь если согласен что все подходы к основаниям математики используют индукцию как одно из неопределяемых понятий???

> Этот пример просто сверх в тему. Это "математическая игра" в её самом идеализированном смысле

Он не в тему потому, что бесполезен в наших вопросах. Определение ФС все тут и так знают, зачем ты притащил какую-то конечную её реализацию, если факт её существования ничего не доказывает? Она ПРИНЦИПИАЛЬНО не может ответить на вопрос определения N, так как не может перейти на мета-уровень.

> Ты просто отбеливаешь философию, называя её "основами математики".

Нет, это ты юзаешь гнилой приёмчик пытаясь спихнуть неудобные вопросы в другую область. Это не ново, кажется в 19 (или 20, не помню) веке какой-то математик считал парадоксы лингвистической проблемой а не математической. Тем не менее из-за этих "лингвистических" проблем всю математаику перекроили и где сейчас этот математик со своим эпичным пердежом в лужу???

> Не нужно, ровно как мне не нужно содержательное понимание "понимания", которое предоставляет нейронауки, чтобы заниматься математикой.

Это уже дешёвая демагогия пошла. Я же не требую от тебя дать определение неопределяемых понятий. Я лишь говорю что нужно навести в них порядок, договориться какое понятие мы считаем интуитивно понимаемым, а какое определяем через другие.

> Чтобы понимать такие вещи, не нужно иметь за плечами монографию по мат. логике.

Это частные случаи конкретных ФС, я не понимаю что ты хочешь ими показать приводя в пример.

> Видимо ты имеешь в виду "естественный язык" (который я действительно встречал в литературе по мат.логике), но надо чётко понимать, что это не математический термин и не математический объект, а значит про него нельзя делать никаких математических утверждений.

Никто и не требует развивать на этот счет математическую теорию или что-то там формализовать. Но если ты в метаязыке юзаешь какое-либо понятие, будь добр за него ответить.

> Ничего не могу посоветовать кроме того, чтобы перечитать цитату из бурбаков.

А я тебе рекомендую перечитать что такое финитный метод. Ты похоже его так и не усвоил судя по тому как пытаешься тут описать формализацию по Гильберту.

> Не видел ни цитат, ни (математических) рассуждений. (Но приводить их лично мне не надо, по причинам, изложенным ниже).

Мдэ, тяжелый случай. Т.е. ты спрашиваешь цитаты математиков, а когда их тебе предъявляют говоришь что не хочешь их читать...

Рассуждение предельно простое: для определения ФС нужные понятийные средства уровня интуитивного определения N, а значит последующее рассмотрение ФС натуральных чисел не могут рассматриваться как их определение.

> Отвечу вопросом на вопрос. Какова будет ценность всех шахматных партий, если изложить правила шахмат, пользуясь средствами противоречивой теории множеств? Ответ: ровно такая же, как и была до этого.

Ответ конечно же не верный. Во-первых, как я уже писал про аксиомы геометрии, это тоже слишком простой пример, так как понятия объектной и мета теории лежат в абсолютно разных плоскостях. Во-вторых, он очевидно неявно полагается на тот факт что эта ФС в силу своей простоты уже изучена другими примитивными методами и про нее заведомо известно что она не противоречива - отними этот факт и тебя бы любой математик послал с такой ФС, так как ценность их в таком случае нулевая.

> Вероятно, это моё последнее подобное сообщение, ибо диалог пошёл по второму кругу, а продвижений никаких.

Ну так кто виноват, что ты аргументов не приводишь? Если ты сто раз скажешь одну и ту же бездоказательную херню, истиной она от этого не станет.

В таком случае тебе будет засчитан слив, и все будут знать что твой ник означает человека не сумевшего определить N.
Аноним 28/09/15 Пнд 15:32:05  301192
>>301190

Вот одна из цитат Александрова (не путать с пидорком П.С. Александровым), даю её отдельно так как пост оказался выше лимита сосаки:

Определяют ли аксиомы Пеано натуральные числа? Нет. Как писал сам Пеано, «число (целое положительное) невозможно определить (ввиду того, что идеи порядка, следования, агрегирования и т. д. столь же сложны, как идея числа)». Иными словами, если вы не знаете, что это такое, так аксиомы Пеано вам не помогут. Вместо этого они описывают имеющуюся у нас идею числа, формализуют наши представления об арифметике, давая возможность строить доказательства.
Аноним 28/09/15 Пнд 15:33:54  301193
>>299871

> считает что это приводит к "порочным кругам" (в чём не прав нисколько)

Какие ваши доказательства???
Аноним 28/09/15 Пнд 16:34:19  301205
>>300843

Я имел ввиду "Доказуемое и не"
Аноним 28/09/15 Пнд 16:54:04  301211
(27Кб, 421x604)
Ще не вмерла математика в Россиюшке. 250 постов вдумчивого обсуждения аксиомы. Горжусь сосачем!
Аноним 28/09/15 Пнд 17:44:00  301213
>>301211
Так ты открой dxdy, там даже раздел для доказателей теоремы Ферма есть! Потому что могут!
Аноним 28/09/15 Пнд 20:13:01  301272
>>301213

Arsenij Knuebook???
knuebok 28/09/15 Пнд 20:47:15  301283
>>301272
ne ya, I ya "knuebok".
Аноним 28/09/15 Пнд 22:10:16  301313
>>301283

knuebok cannot into natural numbers.
knuebok 28/09/15 Пнд 22:32:39  301323
>>301313
da ti zadolbal, mi prosto v raznih disskursah govoreem, no ti che to etogo ne ponimaesh i prodoljaesh vtirat mne svou dich. skolko mozno uje
Аноним 28/09/15 Пнд 22:37:43  301326
>>301323

no, you ditch
Аноним 28/09/15 Пнд 22:40:31  301327
>>301323

Draining counted.

Аноним 28/09/15 Пнд 22:51:31  301332
Бамп.
Аноним 29/09/15 Втр 09:03:14  301386
Какая гнида подтирает картинки? Рака яиц тебе.
Аноним 29/09/15 Втр 14:24:37  301434
>>301323

Ну ты и пиздло, какие нахуй дискурсы, когда ты несёшь херню. На вот почитай цитату, специально для тебя запостили: >>301192
Аноним 30/09/15 Срд 09:21:18  301589
>>301192
Имхо аксиомы Пеано определяют(формализуют), но не объясняют - как и с любой системой аксиом(возьми хоть аксиомы евклидовой планиметрии, аксиоматика теории множеств, вещественных чисел). Неопределяемые понятия нельзя определить, но можно провести аналогии, дать некие модели, неформально "объяснить".
Аноним 30/09/15 Срд 13:09:38  301646
>>301589
Разумеется, достаточно полная метаинформация об объекте важнее формального определения объекта для исследователя, развивающего теорию.
Аноним 30/09/15 Срд 13:45:53  301657
>>301438
Расовая черта?
Бамп треду. Если возражений насчёт аксиом Пеано больше нет, то имхо тред заслуживает присуждения статуса платины /sci.
Аноним 30/09/15 Срд 20:12:56  301701
>>301657
У меня есть возражение. Аксиомы Пеано по сути бесполезны, в них не отражены самые важные свойства N. С их помощью нельзя доказать теорему Гудстейна.
Аноним 04/10/15 Вск 02:52:26  302305
>>301701
Я уже не в могу в такой лвл дискуссии, квалификации не хватает-ссс. Узнал про теорему Гудстейна с твоих слов - но попытаюсь возразить - хватает же арифметики первого порядка в плане приложений(матана, линейки, даже ТЧ и прочаго) - зачем нам какие-то хитровыстраданные теоремы на арифметике 2-го лвла?
Аноним 08/10/15 Чтв 20:18:05  303228
>>302217
...Математик отошел вправо. Математик наклонился. Математик вложил себе в рот четыре пальца. Математик завёл натуральные числа через аксиоматику Пеано. Математик вложил себе в рот два пальца. Математик завёл натуральные числа через аксиоматику фон Неймана. Математик вложил себе в рот четыре пальца. Математик завёл натуральные числа через аксиоматику Пеано. Математик опустился на колени. Математик стал сгребать натуральные числа в кучу. Математик сгреб натуральные числа в кучу. Математик присел над кучей. Математик ввёл кванторы в кучу натуральных чисел. Математик стал перемешивать кванторы с натуральными числами. Математик перемешал кванторы с натуральными числами. Математик зачерпнул пригоршню смеси. Математик намазал смесью кванторов и натуральных чисел свой член. Математик встал на колени. Математик стал мастурбировать. Математик доказал коммутативность сложения. Математик лег на спину. Математик стал пальцами доказывать коммутативность умножения. Математик доказал коммутативность умножения. Математик лег на живот. Математик подполз к смеси натуральных чисел с кванторами. Математик стал записывать в TeXе смесь. Математик записал всю смесь. Математик стал вылизывать участок пола на месте смеси. Математик вылизал участок пола. Математик встал с пола. Математик подпрыгнул. Математик похлопал себя ладонями по щекам. Математик надул щеки. Математик хлопнул себя ладонями по щекам. Математик надул щеки. Математик хлопнул себя ладонями по щекам. Математик наклонился. Математик ввёл себе эпсилон-дельта-формализм в задний проход. Математик вынул формализм из заднего прохода. Математик ввёл эпсилон-дельта-формализм себе в правую ноздрю. Математик облизал эпсилон-дельта-формализм. Математик присел на корточки. Математик налил водовки себе в ладонь. Математик вылил свою водовку себе на голову. Математик сел на пол. Математик разглядывал и трогал свои ноги. Математик зачерпнул меры Хаара с пола. Математик конструктивно ввёл меру Хаара себе на ноги. Математик разнёс меру Хаара левыми сдвигами. Математик согнул ноги в коленях. Математик качал ногами. Математик лег на спину. Математик вытянул ноги. Математик поднял ноги. Математик запрокинул ноги себе за голову. Математик стал элиминировать кванторы. Математик элиминировал себе на лицо...
Аноним 16/10/15 Птн 23:07:48  304682
/\,\/,->,-,∀x,∃x
F->(H->F)
(F->(G->H))->((F->G)->(F->H))
F/\G->F
F/\G->G
F->(G->F/\G)
F->F\/G
G->F\/G
(F->G)->((H->G)->(F\/H->G))
-F->(F->G)
(F->G)->((F->-G)->-F)
F\/-F
(∀x. F(x))->F(t)
F(t)->(∃x. F(x))


F F->G
----------
G

F
----------
∀x. F

N,=,0,S,+,*
∀x∈N. 0 ≠ S(x)
∀x,y∈N. S(x) = S(y) ⇒ x = y
∀x∈N. x + 0 = x
∀x,y∈N. x + S(y) = S(x + y)
∀x∈N. x ⋅ 0 = 0
∀x,y∈N. x ⋅ S(y) = x ⋅ y + x
∀y∈N(F(0,y) -> (∀x∈N. F(x,y) -> F(S(x),y)) -> ∀x∈N. F(x,y))

Аноним 20/10/15 Втр 20:02:09  305754
(60Кб, 790x593)
Призываю пикрилейтед в тред, да и мимо-диванам с аналитикой буду рад. N-тред, живи!
Автору маскота треда отдельное спасибо.
Аноним 20/10/15 Втр 20:04:51  305757
(46Кб, 460x592)
Аноним 21/10/15 Срд 14:02:02  305890
В википедии написано, чтл аксиомы Пеано определяют N, а вы говорите, что не определяют. И кому же мне блядь поверить? Анонимным кукаретикам с форума про мемесы, или статье в Википедии, где приведены ссылки на авторитетные источники и которая модерируется настоящими дипломированными математиками? Я даже блядь не знаю.
Аноним 21/10/15 Срд 19:06:48  306026
>>305890
Ad verecundiam.
Аноним 21/10/15 Срд 19:37:03  306032
>>305890
Что ты местных долбаебов слушаешь. Аксиомы Пеано определяют натуральные числа.
Аноним 21/10/15 Срд 20:18:38  306052
>>305890
>Википедия
>модерируется настоящими дипломированными математиками
Да ну, правда что ли?
Аноним 22/10/15 Чтв 00:28:00  306137
Где мои посты?
knuebok 22/10/15 Чтв 00:42:24  306139
>>306052
Не то, чтобы модерируется, но проходят экспертную оценку, статьи, которые её не прошли заносятся в эту категорию https://en.wikipedia.org/wiki/Category:Mathematics_articles_needing_expert_attention . Всё сказанное верно только для энвики, разумеется.
Аноним 22/10/15 Чтв 01:12:54  306141
>>305890
>>306139
Вот вам мнение чуть более авторитетного форума про мемесы для дипломированных математиков
http://mathoverflow.net/questions/47399/dont-the-axioms-of-set-theory-implicitly-assume-numbers
knuebok 22/10/15 Чтв 03:25:42  306151
>>306141
Прекрасная ссылка! В самом популярном ответе написано то, что я тебе пытался объяснить на протяжении (двух?) месяцев:

Logical foundations avoids this paradox ultimately by being concrete. We may put it this way: logic at the primary level consists of instructions for dealing with formal linguistic items, but the concrete actions for executing those instructions (electrons moving through logic gates, a person unconsciously making an inference in response to a situation) are not themselves linguistic items, not of the language. They are nevertheless as precise as one could wish.

We emphasize this point because in our descriptions below, we obviously must use language to describe logic, and some of this language will look just like the formal mathematics that logic is supposed to be prior to. Nevertheless, the apparent circularity should be considered spurious: it is assumed that the programmer who reads an instruction such as "concatenate a list of lists into a master list" does not need to have mathematical language to formalize this, but will be able to translate this directly into actions performed in the real world. However, at the same time, the mathematically literate reader may like having a mathematical meta-layer in which to understand the instructions. The presence of this meta-level should not be a source of confusion, leading one to think we are pulling a circular "fast one".
In other words, to break out of the circularity,it is enough to observe that computers can be programmed to recognize certain strings as well-formed terms or formulas (of a given axiomatic theory), and how to recognize inferences as valid.

It's not as if there needs to be some background theory, or the prior existence of a completed or actual infinity of all expressions which might come up, sitting inside the computer. The computer is programmed to handle finite parts of the theory correctly, and the same applies to human users of a theory (although we say "taught", not "programmed").

>but the concrete actions for executing those instructions (electrons moving through logic gates, a person unconsciously making an inference in response to a situation) are not themselves linguistic items, not of the language
То есть абсолютно то, что я говорил: математик играет в синтаксическую игру, а то, умеет ли он распознавать символы и использует ли он при их распознавании априорную философскую идею о натуральных числах - никого не тревожит, это не часть математики, не часть правил игры. Кстати, даже аналогии про компьютерные программы я похожие приводил, когда давал ссылки на metamth.
knuebok 22/10/15 Чтв 04:43:43  306153
Мне настолько понравился ответ, что я решил перевести (английский плохо знаю, перевод кривенький и в некоторых местах слишком вольный, и вообще сейчас 5 утра, поэтому все улучшения перевода приветствуются):

Основания математики избегают этого парадокса, будучи вполне конкретными. Мы можем думать об этом следующим образом: логика на самом фундаментальном уровне содержит инструкции, для преобразования некоторых формальных лингвистических объектов. Но конкретные действия, для выполнения этих инструкций (движение электронов по микросхемам, или подсознательные умозаключения, которые мы должны сделать, чтобы выполнить данные инструкции) сами по себе не являются лингвистическими объектами, не являются частью языка.

Мы заостряем внимание на этом моменте, потому что в наших дальнейших построениях мы, очевидно, должны использовать естественный язык, чтобы описать логику, и некоторые части этого естественного языка будут выглядеть для нас прямо как формальная математика, основания которой мы и пытаемся построить. Тем не менее, соответствующий "порочный круг" следует считать недействительным, так как мы предполагаем, что программист, который читает инструкцию типа "объедините два списка в один" не нуждается в математическом языке для формализации этого действия, он должен лишь уметь это непосредственно исполнить. Тем не менее, математически подготовленный читатель может предпочесть иметь "метатеорию" в которой он будет "понимать" эти инструкции. Существование этой "метатеории" не должно рассматриваться как источник противоречий, можете думать об этом не как о "порочном круге" а как о "спирали" которую мы "прокручиваем" всего один раз.

Другими словами, чтобы разорвать "порочный круг", достаточно заметить, что можно запрограммировать компьютер, распознавать конкретные строки как корректные термы и формулы (данной аксиоматической теории), и отличать верный вывод от неверного. Этому компьютеру совсем не нужна фоновая теория, или существование актуальной бесконечной совокупности всех формул, которые он мог бы распознать. Компьютерная программа нужна лишь для того, чтобы распознавать конечные части теории корректно, то же самое касается и непосредственных пользователей теории - людей (хотя мы говорим, что теории "учим", а не "запрограммированы их распознавать").
Аноним 22/10/15 Чтв 10:08:11  306167
>>306153

22 мухи не могут ошибаться??? Кто-то писал про голосование в математике...

Этот ответ не признан решением, к тому же ты умолчал как он писал в начале что имеет место "common-sense circularity" - т.е. ровно тот замкнутый круг между мета и объектной теорией. По сути он не отрицает наличие замкнутого круга, но пытается его как-то легализовать. Вообще его отмазки мне напомнили ситуацию с неграми. Все понимают что это черножопый человек, но нет, явно назвать его негром или черным нельзя (иначе манямирок равноправия разрушится), это АФРОАМЕРИКАНЕЦ. Так и тут. Это замкнутая СПИРАЛЬ, а не круг.

PS Я в упор не понимаю зачем нужна аналогия с компом, она не только бесполезна, но и вредна. Она не добавляет ничего нового - мы и так понимаем что такое формальная система и её "мета-исполнитель". Зачем приводить более сложное понятие компа от которого у нормального человека сразу первая ассоциация будет с конечностью ресурсов машины и теорией алгоритмов которая сама по себе более навороченное понятие чем ФС. К тому же про конкретность проверки он просто врёт. Достаточно взять из начального курса матлогики любую теорему доказанную индуктивно и его комп навернется медным тазом.



Аноним 22/10/15 Чтв 10:09:31  306168
>>306151

> умеет ли он распознавать символы и использует ли он при их распознавании априорную философскую идею о натуральных числах - никого не тревожит

Ну а меня не тревожит, что множество противоречивое понятие и что с того???
Аноним 22/10/15 Чтв 10:47:39  306182
>>306168
То есть тебе похуй на противоречивость? Совсем поехал со своим порочным кругом?
Аноним 22/10/15 Чтв 10:51:06  306187
>>306183
Ты только дураком себя выставил. Что тебе не нравится, скажи максимально подробно и конкретно.
Аноним 22/10/15 Чтв 10:52:57  306188
>>306187

> Ты только дураком себя выставил.

C точки зрения мудака - безусловно.
Аноним 22/10/15 Чтв 10:55:37  306189
>>306188
Пососи мои солёные шоколадные яйца.
Аноним 22/10/15 Чтв 11:44:49  306197
>>306189

Лучше твою мамку нагнуть.
knuebok 22/10/15 Чтв 12:28:16  306201
Я просто повторю то что написано в том, посте, потому что добавить, в принципе, нечего.
>Этот ответ не признан решением, к тому же ты умолчал как он писал в начале что имеет место "common-sense circularity" - т.е. ровно тот замкнутый круг между мета и объектной теорией. По сути он не отрицает наличие замкнутого круга, но пытается его как-то легализовать.
Разумеется, но в том-то и дело что это common-sense circularity, а не math-sense, например, ты вот когда-то агрился на слово "прочувствовать" но по-другому просто нельзя: когда мы начинаем с нуля рассказывать формальную математику на естественном языке, человеку, который ничего о математике не слышал, у нас абсолютно нету никаких математических средств, и хоть и можно помахать руками и сказать что-то там про "финитные средства в смысле Гильберта" - но это всё будет абсолютно неформализуемое махание руками, потому что формализмов, собственно, у нас никаких ещё нету. Почему, например, ты не выписал этот порочный круг явно на протяжении всех этих тредов, то есть не указал предикат phi(x) в формуле которого используется предикат psi(x) и предикат psi(x) в формуле которого используется предикат phi(x)? Потому что никаких предикатов на этом "этапе" ещё нету, и, собственно никакой математики, а значит никаких математических парадоксов - тоже.
А вот после того, как мы объяснили как использовать хоть какую-то теорию (и это объяснения не могло в строгом смысле опираться на математические формализмы, хотя мы и могли использовать квазиматематический язык типа "алфавит - это счётное множество символов", а могли просто сказать "мы можем рисовать таки и такие значки на листочке" - не важно абсолютно), мы уже можем в ней что-то определять в строго математическом смысле и искать в ней порочные круги тоже в строго математическом смысле, но там мы их как раз и не найдём. И, даже более того, все объяснения про то, как пользоваться какой-то теорией - мы уже в этой же теории сможем теперь математически формализовать, но в этом опять же нету никакого порочного круга, мы будем определять по честным правилам честные математические объекты.

С программистами аналогия хорошая очень. В Хаскелле, например, определение натуральных чисел выглядело бы так:
data Nat = Zero | Succ Nat
(0 - это натуральное число,
если n - натуральное число, то Succ(n) - натуральное число,
других натуральных чисел нету)
и ты такой говоришь "но как же так? ведь чтобы написать такое определение уже надо уметь пользоваться Хаскеллем, а значит иметь представление о конечных строках и рекурсивных определениях". Да - надо, но это не проблема Хаскелля, это проблема тех кто его учит, Хаскель будет оставаться по прежнему внутренне непротиворечивой системой и программы на нём по прежнему будут продолжать компилироваться.
Аноним 26/10/15 Пнд 15:08:16  306857
>>306201

> но это всё будет абсолютно неформализуемое махание руками, потому что формализмов, собственно, у нас никаких ещё нету.

И что с того? Это ты с какого-то перепоя требуешь сразу же формализм из ничего. Даже если принять твой подход, то, очевидно, ты просто не имеешь права использовать любое неформальное понятие. Так какого хера ты позволяешь себе это делать? У тебя двоемыслие какое-то, шизик.

> А вот после того, как мы объяснили как использовать хоть какую-то теорию (и это объяснения не могло в строгом смысле опираться на математические формализмы, хотя мы и могли использовать квазиматематический язык типа "алфавит - это счётное множество символов", а могли просто сказать "мы можем рисовать таки и такие значки на листочке" - не важно абсолютно), мы уже можем в ней что-то определять

По сути тебе надо определить счетное множество таких значков. Вот тебе и готовое интуитивное N по Маркову.

Такое неформальное "объяснение" как раз и есть интуитивное понимание - разъяснение неопределяемого понятия на серии примеров и образов реального мира.

> но в этом опять же нету никакого порочного круга, мы будем определять по честным правилам честные математические объекты.

Пример ЧЕСТНОГО определения N без интуитивной метаиндукции или пиздабол.

> С программистами аналогия хорошая очень. В Хаскелле, например, определение натуральных чисел выглядело бы так:

Ну зачем было так пафосно выёбываться, если ты в итоге написал тривиальное индуктивное определение? Это не аналогия, а частная реализация формальной системы в железе (или описание абстрактной машины, если ты это имел ввиду).

> Да - надо, но это не проблема Хаскелля, это проблема тех кто его учит, Хаскель будет оставаться по прежнему внутренне непротиворечивой системой и программы на нём по прежнему будут продолжать компилироваться.

Они будут компилироваться только лишь потому что есть априорное понимание метаиндукции. Компы это вещь которая разработана и работает на основе ФС, а не наоборот.

PS Кажется я понял откуда растёт хвост этой поеботины с компом. В книжках по матлогике когда излагаются суть финитного метода часто говорят о его конструктивности. Это собственно и есть оправдание почему финитный метод считается "хорошим" в отличии от наивного канторовского и дает веру в отсутствие противоречий (ну и попутно решает задачу минимальности исходных понятий). Но он не то что не отвергает интуитивность метаиндукции, он как раз её пытается обосновать на неформальном уровне...
Так вот, похоже говоря про комп котороый сам строит и перебирает строки тот мужик пытался современным языком подчеркнуть её конструктивность, а кнуёбок как обычно всё перепутал со своими дискурсами.
Аноним 26/10/15 Пнд 15:29:46  306864
>>306857
Я готов дать тебе определение N и разговаривать с тобой сколь угодно долго. Но сперва скажи, что такое интуитивная метаиндукция?
Аноним 26/10/15 Пнд 15:35:42  306865
>>306864

Интуитивное понимание индукции на естественном метаязыке.
Аноним 26/10/15 Пнд 15:44:15  306867
>>306865
Да. Что это такое?
Аноним 26/10/15 Пнд 15:47:16  306869
>>306867

Неопределяемое понятие.
Аноним 26/10/15 Пнд 15:49:50  306870
>>306869
Тогда каким же образом ты различаешь, где есть интуитивная метаиндукция, а где она отсутствует?
Аноним 26/10/15 Пнд 15:55:10  306871
>>306870

Распутываю цепочку определений пока не найду.
Аноним 26/10/15 Пнд 15:56:03  306872
inb4: метаиндукция есть? а если найду???
Аноним 26/10/15 Пнд 16:11:54  306873
>>306871
Ну а как ты понимаешь, что нашёл? Должны же у тебя быть какие-то критерии.
Аноним 26/10/15 Пнд 16:25:44  306874
>>306873

Как только нахожу явное использование метаиндукции так сразу даю себе команду останова.
Аноним 26/10/15 Пнд 16:46:33  306876
>>306874
Научи меня находить метаиндукцию. Я хочу быть как ты.
Аноним 26/10/15 Пнд 16:47:52  306877
Такс-такс-такс, шо тут у нас, N-петух опять всрався, не надоело ещё?
Аноним 26/10/15 Пнд 16:49:27  306878
>>306877

> всрався

Что?
Аноним 26/10/15 Пнд 16:50:27  306879
>>306878
Интуитивная метаиндукция через плечо, МРАЗЬ.
Аноним 26/10/15 Пнд 16:51:52  306880
>>306879

Ты - ПИЗДЛО.
Аноним 27/10/15 Втр 00:36:38  306983
>Так вот, похоже говоря про комп котороый сам строит и перебирает строки тот мужик пытался современным языком подчеркнуть её конструктивность, а кнуёбок как обычно всё перепутал со своими дискурсами.
Смешные маняврирования N-петуха, который даже в том факте, что его мамка - шлюха, увидит подтверждение своей позиции. Для тебя фраза
>Logical foundations avoids this paradox ultimately by being concrete.
"того мужика" недостаточно однозначна, ммм, ПИЗДЛО МРАЗЬ?
Аноним 27/10/15 Втр 01:51:15  306996
>>295522
Почему это мне так сильно напоминает лямбда-термы? Олсо, про определение натуральных чисел через лямбды, а индукции через Y-комбинатор уже говорили?
нубас в этой теме, по лямбдам всего одна лекция была, зато какая!
Аноним 27/10/15 Втр 09:29:07  307033
>>306983

Ты хуйло пиздливое, иди учись читать по англицки, ПИЗДЛО малограмотное!
Аноним 27/10/15 Втр 09:30:38  307034
>>306983

Давай, хуйло пизливое, объясни каким образом конкретность конечного числа элементов без абстракции потенциальной осуществимости может решить этот парадокс?
Аноним 27/10/15 Втр 09:51:09  307041
>>306874
Топовые маневры N петушка
sage 27/10/15 Втр 14:16:25  307100
>>307034
Могу ответить только:
>Ты хуйло пиздливое, иди учись читать по англицки, ПИЗДЛО малограмотное!
Аноним 27/10/15 Втр 14:40:56  307106
>>307100

Ты малограмотное пиздло. А твоя мать жирная похотливая шлюха.
Аноним 27/10/15 Втр 14:41:24  307108
>>307100

> Могу ответить только

Это характерно для малолетнего долбоёба.
Аноним 27/10/15 Втр 14:52:20  307110
>>307106
>>307108
N-петух, а у тебя бомбит настолько, что ты на каждый мой пост дважды в течении двух минут отвечаешь? Просто тут: >>307033 >>306996 тоже так было, то есть твой пердак полыхает настолько, что ты не можешь остановиться отвечать мне?
Аноним 27/10/15 Втр 15:00:59  307111
>>307110

Просто вторая мысль пришла чуть позже.

Да и вообще, разве сложно пару раз послать тебя нахуй???
Аноним 27/10/15 Втр 15:03:22  307112
>>307111
Глубокие у тебя мысли "чуть позже" приходят, почти что на уровне интуитивной метаиндукции, а быть может даже и выше.
Аноним 27/10/15 Втр 15:05:38  307113
>>307112

C точки зрения мудака - безусловно.
Аноним 27/10/15 Втр 15:06:10  307114
>>307113
С точки зрения мудака - безусловно.
Аноним 27/10/15 Втр 15:06:56  307116
>>307114

>>307113
Аноним 27/10/15 Втр 15:07:52  307117
>>307116

>>307114
Аноним 27/10/15 Втр 15:08:41  307118
>>307117

>>307114
Аноним 27/10/15 Втр 15:15:16  307120
>>307118
У нас прямо интуитивная метаиндукция сейчас выходит, чего же ты не даёшь самому себе команду останова?
Аноним 27/10/15 Втр 15:21:48  307121
>>307120

Пару раз проверил чтобы убедиться на 100%, теперь даю.
Аноним 27/10/15 Втр 15:23:31  307122
А я не даю.
>>307118

>>307114
Аноним 27/10/15 Втр 15:26:42  307124
>>307122

Так ты же кнуёбок, ты не умеешь в неопределяемые понятия.
Аноним 27/10/15 Втр 15:28:02  307125
>>307124
У тебя кнуёбкофобия.
Аноним 27/10/15 Втр 15:29:40  307127
>>307125

кнуёбок, залогинься.
кнуёбок 27/10/15 Втр 15:31:47  307128
>>307127
Залогинился.
Аноним 31/10/15 Суб 17:37:47  308406
Ну так что?
Аноним 17/11/15 Втр 17:03:30  312062
>>312055
Цикличность действительно существует, но индукция здесь не причём.
- Аксиома индукции ссылается на N.
- N формализуется аксиоматикой Пеано.
- Используя аксиому индукции, доказывается, что N ⊆ {0, s(0), s(s(0)), ...}, а следовательно N = {0, s(0), s(s(0)), ...}

А вот это уже по-моему грустно. Фактически имея цикличность и используя нами же написанную аксиому мы показываем равенство объектов N и {0, s(0), s(s(0)), ...}.

В чём я неправ?
Аноним 17/11/15 Втр 17:19:57  312066
>>312062

Так ты прав.
Аноним 17/11/15 Втр 17:30:26  312073
>>312066
В каком месте он прав?
Аноним 18/11/15 Срд 15:07:02  312234
>>312073
В чём-то не прав.
Аноним 18/11/15 Срд 20:08:55  312374
В мат треде мне не отвечают. Поясните, зачем нужно определять $\mathbb{N}$? Если в какой-то теории нельзя определить $\mathbb{N}$ или не выводятся элементарные свойства $\mathbb{N}$ (например те, которые описаны в аксиомах Пеано) или выводятся их отрицания, то эта теория бесполезна. Почему бы не считать натуральное число неопределяемым понятием? Множество же не определяют, а натуральное число интуитивно понятнее множества. Под "интуитивно понятнее" я имею в виду, что, скажем, средний детсадовец научится распознавать натуральные числа среди других математических объектов быстрее чем множества.
Аноним 19/11/15 Чтв 05:02:06  312484
>>312374
Затем что аксиоматика Пеано используется в различных областях математики среди которых теория чисел, теория категорий и алгебра.
Аноним 30/11/15 Пнд 18:53:15  315013
...Математик отошел вправо. Математик наклонился. Математик вложил себе в рот четыре пальца. Математик завёл натуральные числа через аксиоматику Пеано. Математик вложил себе в рот два пальца. Математик завёл натуральные числа через аксиоматику фон Неймана. Математик вложил себе в рот четыре пальца. Математик завёл натуральные числа через аксиоматику Пеано. Математик опустился на колени. Математик стал сгребать натуральные числа в кучу. Математик сгреб натуральные числа в кучу. Математик присел над кучей. Математик ввёл кванторы в кучу натуральных чисел. Математик стал перемешивать кванторы с натуральными числами. Математик перемешал кванторы с натуральными числами. Математик зачерпнул пригоршню смеси. Математик намазал смесью кванторов и натуральных чисел свой член. Математик встал на колени. Математик стал мастурбировать. Математик доказал коммутативность сложения. Математик лег на спину. Математик стал пальцами доказывать коммутативность умножения. Математик доказал коммутативность умножения. Математик лег на живот. Математик подполз к смеси натуральных чисел с кванторами. Математик стал записывать в TeXе смесь. Математик записал всю смесь. Математик стал вылизывать участок пола на месте смеси. Математик вылизал участок пола. Математик встал с пола. Математик подпрыгнул. Математик похлопал себя ладонями по щекам. Математик надул щеки. Математик хлопнул себя ладонями по щекам. Математик надул щеки. Математик хлопнул себя ладонями по щекам. Математик наклонился. Математик ввёл себе эпсилон-дельта-формализм в задний проход. Математик вынул формализм из заднего прохода. Математик ввёл эпсилон-дельта-формализм себе в правую ноздрю. Математик облизал эпсилон-дельта-формализм. Математик присел на корточки. Математик налил водовки себе в ладонь. Математик вылил свою водовку себе на голову. Математик сел на пол. Математик разглядывал и трогал свои ноги. Математик зачерпнул меры Хаара с пола. Математик конструктивно ввёл меру Хаара себе на ноги. Математик разнёс меру Хаара левыми сдвигами. Математик согнул ноги в коленях. Математик качал ногами. Математик лег на спину. Математик вытянул ноги. Математик поднял ноги. Математик запрокинул ноги себе за голову. Математик стал элиминировать кванторы. Математик элиминировал себе на лицо...
Аноним 30/11/15 Пнд 19:39:48  315028
>>315013
У нас уже есть горячие гей пасты с физа про изнасилование в раздевалке зала, а теперь еще и математик-вован N-петух.
Аноним 30/11/15 Пнд 20:40:24  315043
>>295737
Кек, я тоже так могу.
1, 2, 3, 4... ну вы понели
Аноним 24/12/15 Чтв 09:11:26  320946
>>315043

Что такое "..."?
Аноним 24/12/15 Чтв 13:36:36  320994
N = 0
/tred
Аноним 25/12/15 Птн 03:29:12  321237
>>320946
1. С общепринятой точки зрения, определение таково:
(...)=(...)[конкатенация].
Т.е. это как прибавить единицу но на один больше.
2. С точки зрения N-петуха: Метаиндукция, Априорное знание, Самореференция
Аноним 25/12/15 Птн 04:50:12  321246
Эй, Н-петушара, а что такое "определить"?
Аноним 18/01/16 Пнд 15:25:15  329750
>>321246
Бамп, хули.
Аноним 18/01/16 Пнд 15:30:13  329755
>>321237

> Метаиндукция, Априорное знание, Самореференция

И что тут не так???
Аноним 19/01/16 Втр 19:20:39  330404
Почему мой пост удалили? Ерунда какая-то.

Мне действительно печально видеть, что два специалиста не могут понять друг друга. Кнуебку говорили об одном, а он в ответ метал пассажи об очевидных, в общем-то, вещах, которые не имели никакого отношения к сути претензий Эн-петуха.

Проблема тут, как мне кажется, в отсутствии достаточной широты знаний, всестороннеразвитости, если хотите. Подняться на уровень выше бывает очень сложно, потому как парадигма мышления устоялась и воспринимается (некоторыми) как нечто незыблемое. Впрочем, это все тоже очевидные вещи, specialization is for insects, вот это все.

Модератор, пожалуйста, не удаляй вот так посты.

Во-первых, это бессмысленно: я ведь не понял, что именно тебе в нем не понравилось, почему ты его удалил и как избежать этого в будущем.

Во-вторых, это обидно: я потратил некоторое количество своего времени, чтобы написать пост, а ты удаляешь без видимых причин любое упоминание о нем.

Пожалуйста, не делай так. Вырази каким-либо образом свои претензии к посту, прежде чем его удалять. Давайте будем конструктивными.
Аноним 21/02/16 Вск 08:21:39  340430
Едет тестер программного обеспечения с работы домой в метро. Едет на метро. Едет в первом вагоне. Едет в нулевом вагоне. Едет в -1 вагоне. Едет между вагонами. Едет в бесконечном количестве вагонов.
Едет в половине вагона. Едет в трех вагонах. Едет без вагона. Ведет вагон. Ведет втроем. Ведет наполовину. Едет в fjugdjhfdjhfajhafhjajfhfakjfa. Едет и неистово .
Аноним 21/02/16 Вск 08:42:45  340432
>>340430
>Едет в бесконечном количестве вагонов.
>тестер программного обеспечения
Fial
Аноним 21/02/16 Вск 10:28:32  340438
>>340432
Бесконечность равна INT_MAX, UINT_MAX, LONG_MAX, ULONG_MAX, LLONG_MAX, ULLONG_MAX и описана в limits.h.
Аноним 21/02/16 Вск 10:42:15  340439
Тред не читал, но насколько я понимаю вся проблема определения N состоит в трудностях определения еденицы. Почему нельзя определить единицу так:
Единица это отношение сторон квадрата
И все, тред можно закрывать.
Аноним 21/02/16 Вск 11:05:12  340442
>>340439
>думать, что этот форсотред существует для заявленных в оппосте целей.
Аноним 21/02/16 Вск 12:23:02  340453
>>340438
"А мужики то не знают."
Аноним 21/02/16 Вск 19:11:13  340580
>>340440
А где я про числа говорил? 2 стороны квадрата это по твоему числа?
Аноним 22/02/16 Пнд 09:50:47  340699
Аксиомы Пеано придумал Пеано. А кто такой был Пеано?
Аноним 22/02/16 Пнд 14:54:36  340755
N-обиженный, а почему ты не прицепился к аксиомам Евклида, ведь там есть, к примеру формулировка
Через 2 точки проходит одна единственная прямая. - Т.к. аксиомы ты считаешь "определением", получается что в определении объекта мы ссылаемся на сам объект. СамореференциЁ же!
С индукцией та же фигня - ей нужны только понятия базы(1), функции следования;
Высказывания, зависящего от элемента N, высказывания верного для любого N. Последние 2 компонента не требуют знания о внутреннем устройстве элемента N или множества N, для нас это суть формальные термы предиката, их внутренняя структура нас не интересует.
Аноним 24/02/16 Срд 12:57:50  341270
Пацаны, смотриче чё принес: единица - это элементарная функция нейрона. Математические объекты приравниваем к объектам восприятия. На вопрос, что такое нейрон и восприятие - шлем к биологам. Классно придумал?
Аноним 24/02/16 Срд 16:03:21  341306
Я не понимаю, почему нельзя определить единицу, как отношение сторон квадрата. Есть понятие "величины" в математике. Отношение двух равных величин это единица. Все. ВСЕ БЛЯДЬ.
Аноним 24/02/16 Срд 16:31:24  341314
(159Кб, 3450x2550)
>>341306
Кстати, да. У тех же бурбаков понятие равенства и его свойства достаточно строго определено. Берем теорему 1 пикрелейтед (х = х), и выводим из нее следующую, х = х, х/х = 1. Дальше аксиомы Пеано. Вот и определили N.
Аноним 24/02/16 Срд 16:36:36  341319
>>341317
>Отношения определены над числами, долбаёб.
Или над элементами множеств. Соснул ты.
Аноним 24/02/16 Срд 16:45:19  341322
(26Кб, 475x140)
>>341320
Запросто, клован. Ни числа, ни его свойств для определения множества не требуется в принципе. Иди Пеано во сне смотри, обиженный))
Аноним 24/02/16 Срд 16:58:08  341332
>>341330
Еблан, у бурбаков в первом томе про теорию множеств вообще понятия числа не используется.
Аноним 24/02/16 Срд 17:02:09  341337
>>341320
Долбаеб это ты, понятие "множества" в математике неопределяемо. Вот пруф
https://ru.wikipedia.org/wiki/Множество
>онятие множества обычно принимается за одно из исходных (аксиоматических) понятий, то есть несводимое к другим понятиям, а значит, и не имеющее определения
Аноним 24/02/16 Срд 17:05:37  341342
>>341338
Я понимаю следствия теорем Геделя. Невозможно доказать непротиворечивость арифметики Пеано.
Аноним 24/02/16 Срд 17:10:48  341346
>>341343
Во-первых, Гедель доказал, что любой набор правил вывода, способный правильно обосновать даже доказательства обычной арифметики, никогда не сможет обосновать доказательство своей собственной согласованности. Следовательно, нечего и надеяться найти доказуемо согласованный набор правил, который предвидел Гильберт. Во-вторых, Гедель доказал, что если какой-то набор правил вывода в некоторой (достаточно обширной) области математики является согласованным (неважно, доказуемо это или нет), то в пределах этой области должны существовать обоснованные методы доказательства, которые эти правила не могут определить как обоснованные.
Аноним 24/02/16 Срд 17:15:07  341349
(61Кб, 807x385)
Все карочи мы апределили еденицу как отношения двух равных виличин, N-петух соснул, Слава Геделю!
Аноним 24/02/16 Срд 17:17:26  341354
>>341348
Для систем, описанных в логике первого порядка, сильно аксиоматизированных.
Аноним 24/02/16 Срд 17:17:36  341355
>>341349
Объясни, почему это не определение единицы.
Аноним 24/02/16 Срд 17:31:07  341363
Все посаны, меня затралели. У меня просто разрывается пукан. Я говорю N-петуху - единица определяется как отношение двух равных величин. А он такой: нихуя это не так, но я тебе не поясню почему это не так, ты тупой. Ну вот как с ним дискуссии вести?
Аноним 24/02/16 Срд 17:41:34  341366
(11Кб, 1058x119)
О чем вообще спор, аутисты, если единица у бурбаков формализована?
Аноним 24/02/16 Срд 17:46:01  341370
>>341366
А ты понимаешь вообще что тут написано? Разъяснить можешь? Или просто копипасту сюда вбросил, даже не чдумываясь?
Аноним 24/02/16 Срд 18:13:21  341381
>>341370
Вот ты и слился. "Ваши пруфы - не пруфы", да? Суть в том, что единицу можно определить без определения числа как понятия. Только использованием теории множеств. Итого, единица определена, исходя из определения единицы все остальные натуральные числа выводятся в аксиоматике Пеано. Все, свободен, иди спи, может быть Пеано приснится))
Аноним 24/02/16 Срд 20:11:19  341419
>>341408
А число и есть йоба-абстракция, прикинь. Единица - это что? Есть один шекель, один трап и один монгол. Что у них общего? То, что их по одному. Это и есть абстракция, тогда как например один шекель - это частный случай. Фишка в том, что с единицей можно манипулировать, не привязываясь к конкретному содержанию. И результат будет одинаковый для любого частного случая. Это и есть суть. Тогда как вот эта шизофазия >>341410>>341414>>341418 просто нежелание пить прописанные таблетки. Мудаку сто раз все разложили, а он все свою хуйлософию гонит.
Аноним 24/02/16 Срд 21:17:15  341454
Вы меня уже заебали. Единица это неопределяемое понятие, это фундамент который мы фиксируем для дальнейшего определения N. Закрывайте нахуй этот тред.
мимо Миша Вербицкий
Аноним 26/02/16 Птн 15:16:38  341973
>>340755

> Через 2 точки проходит одна единственная прямая. - Т.к. аксиомы ты считаешь "определением", получается что в определении объекта мы ссылаемся на сам объект. СамореференциЁ же!

Для аксиом планиметрии можно построить интерпретацию без интуитивного представления о прямых и точках. Для натуральных чисел это невозможно сделать.

> С индукцией та же фигня - ей нужны только понятия базы(1), функции следования;

Функция следования это и есть форма индукции.

Аноним 26/02/16 Птн 15:26:42  341974
>>341454

Пруфы что ты миша?
Аноним 26/02/16 Птн 15:28:49  341975
>>341381

> Итого, единица определена, исходя из определения единицы все остальные натуральные числа выводятся в аксиоматике Пеано.

Нет, для того что бы сознание смогло применить аксиомы Пеано ему априори требуется на уровне интуиции понимание индукции.
Аноним 26/02/16 Птн 15:55:42  341981
>>341975
>для того что бы сознание смогло применить аксиомы Пеано
Фуфлософия. Тебе, еблану, уже сто раз показали, что все сводится к теории множеств, выключай свою шарманку про индукцию.
Аноним 26/02/16 Птн 16:04:53  341982
>>341981

А наивная теория множеств это тоже по-твоему филосософия, ась?
Аноним 26/02/16 Птн 16:09:55  341984
>>341981

Так вы не юзайте философское понятие индукции, я против что ли? Проблема в том, что вы всегда это делаете когда пытаетеcь свести N к теории множеств.
Аноним 26/02/16 Птн 16:40:36  341993
>>341984
>Так вы не юзайте философское понятие индукции,
Так уже. Тебе сто раз показано, как бурбаки вывели единицу без всякой твоей индукции.
Аноним 26/02/16 Птн 17:05:01  341997
>>341993

Так вы выдергиваете куски цитат, опуская все что привело к ним. А там, у Бурбаки, до этого определения предполагается что уже известна логика формальных систем, где уже есть интуитивная индукция.
Аноним 26/02/16 Птн 17:29:37  342000
>>341997
>Так вы выдергиваете куски цитат, опуская все что привело к ним.
Нет никаких цитат, там формула. Аксиомы Пеано так же записаны формулами. Соснул ты, признай уже.
Аноним 26/02/16 Птн 18:21:06  342010
>>342000

Любая формула (так же как и аксиомы Пеано) не может висеть в воздухе, это элемент формальной системы. А теория ФС не может обойтись без индукции. У бурбаки теория ФС вообще считается как пререквизит и не описана.
Аноним 26/02/16 Птн 19:22:31  342041
>>342010
>Любая формула (так же как и аксиомы Пеано) не может висеть в воздухе, это элемент формальной системы.
Что сводится к исчислению предикатов на основе аксиом ZFC. Где там какие фуфлософические индукции?
Аноним 26/02/16 Птн 20:43:56  342060
>>342041

Ты совсем даун что ли? Исчисление предикатов без индукции невозможно определить. Иди читай учебники по матлогу.
Аноним 04/03/16 Птн 15:40:13  344369
>>342041

В абстракции потенциальной бескончности.
Аноним 04/03/16 Птн 18:17:47  344431
>>340718
12 inch peanist.
Аноним 25/03/16 Птн 06:46:54  351191
>>342010
Отбрось гильбертовскую парашу в сторону. Аксиомы Пеано формулируются на естественном языке, например.
Ещё можно определить индукцию до Пеано, а потом просто сослаться на неё.
Аноним 26/03/16 Суб 14:36:29  351488
>>295488 (OP)
Определение? Ты в школе хоть учился?? А не определяют лишь из-за того, что они, по сути, применимы к любым группам, в которых определена функция следования. шах и мат треду
Аноним 26/03/16 Суб 14:43:09  351490
Может вкатит. Хз - определние слегка рекурсивно, но всё же.
N множество нат. чисел (группа по сути) <=>для любого n, принадлежащего N, n^n >=n.
Аноним 26/03/16 Суб 14:44:45  351491
>>351490
Хотя n1^n2>n1. Так точнее
Аноним 27/03/16 Вск 22:04:37  351729
>>351491
при n1 < n2
иначе при n2 = 1 n1^n2 = n1
Аноним 27/03/16 Вск 22:41:27  351737
Натуральные числа определяются за пределами логики, это одно из человеческих чувств, встроенная способность мозга. Что уж там натуральный ряд, вы попробуйте хотя бы true и false формально вывести
Аноним 28/03/16 Пнд 09:45:05  351829
>>351737
Семантика/смысл - это и правда пока "часть человеческого мозга". Формализмы и синтаксис уже как бе освоили. N можно рассматривать как синтактическую конструкцию, не более. "Физический смысл"/априорное знание нам здесь не нужен(но).
Аноним 06/04/16 Срд 05:54:18  353847
>>351737
Теорема Тарского о невыразимости истины.
Аноним 06/04/16 Срд 18:56:43  353957
(28Кб, 445x100)
>>295488 (OP)
>цепочка, посвященная определению N. То есть натуральных чисел.
>почему же аксиомы Пеано не определяют N?

Формальное определение натуральных чисел в 1889 году сформулировал итальянский математик Пеано, основываясь на более ранних построениях Грассмана, в своей книге "Основания арифметики, изложенные новым способом" (лат. Arithmetices principia, nova methodo exposita). В 1888 году (за год до Пеано) практически в точности подобную аксиоматическую систему опубликовал Дедекинд. Непротиворечивость арифметики Пеано доказана (англ.) в 1936 году Генценом с помощью трансфинитной индукции до ординала ε0. Как следует из второй теоремы Гёделя о неполноте, это доказательство не может быть проведено средствами самой арифметики Пеано.
Википедия.

В общем, исходя из этого всего, Пеано смог сформулировать
>определение натуральных чисел.
>То есть натуральных чисел.
А не множества.


Поэтому, удваиваю шары >>353847-куна.
Аноним 07/04/16 Чтв 12:46:47  354098
>>353957
>определение натуральных чисел.
>То есть натуральных чисел.
Проиграл. Счётное и бесконечное множество натуральных чисел и понимается под "натуральными числами" вообще.
Пеано что, определил только несколько(конечное множество) чисел, оставив остальное потомкам? Его аксиомы задают всё их множество N.
Аноним 07/04/16 Чтв 16:06:57  354131
>>354098
Любые два бесконечные множества натуральных чисел равномощны. Не бывает несчётного множества натуральных чисел.
Аноним 07/04/16 Чтв 20:43:25  354172
>>354098
Дать определение элементам множества - не значит определить всё множество.
ИМХО.
Аноним 07/04/16 Чтв 21:01:35  354175
>>354172
Значит. Множество полностью определяется своими элементами.
Аноним 08/04/16 Птн 13:00:24  354287
>>354131
Счётное множество натуральных чисел - масло масляное, это я понимал, т.к. он эквивалентно самому себе и следовательно счётно.
Аноним 08/04/16 Птн 20:48:28  354451
>>354175
Например, подмножествами?
Аноним 09/04/16 Суб 02:06:30  354546
>>354451
Подмножество не есть элемент множества.
Аноним 09/04/16 Суб 06:57:27  354553
>>354546
В общем случае. А так-то иногда вполне может быть и элементом, и подмножеством.
Аноним 09/04/16 Суб 09:49:12  354564
>>354553
Парадокс Рассела.жпг
Аноним 09/04/16 Суб 22:31:25  354750
>>354564
{1, {1}}
Аноним 09/04/16 Суб 22:46:19  354754
>>354750
{,{}}
Аноним 18/04/16 Пнд 18:11:50  356968
>>351191

Как ты сможешь определить индукцию без метаиндукции???
Аноним 18/04/16 Пнд 18:15:20  356970
>>356968
Ну ты же как-то метаиндукцию определил.
Аноним 18/04/16 Пнд 18:15:20  356971
>>353957

Википедия лжет, Пеано дал не определение, а аксиомы которым удовлетворяет множество натуральных чисел. Но без самих натуральных чисал заданных каким-то другим способом, ты эти аксиомы не реализуешь, более того не сможешь их даже сформулировать без опоры на индукцию.
Аноним 18/04/16 Пнд 18:15:52  356972
>>356970

Я не определял, это интуитивное понятие. Т.е. как хошь, так его и понимай.
Аноним 18/04/16 Пнд 18:18:34  356973
>>356972
То есть метаиндукция - это пингвин?!
Аноним 18/04/16 Пнд 18:23:03  356974
>>356973

Если пингвин это метаиндукция, то да.
Аноним 18/04/16 Пнд 18:27:17  356976
(362Кб, 262x219)
>>356971
Шизик, сука, тебе сколько раз еще повторять, что формальное определение единицы есть у бурбаков?
Аноним 18/04/16 Пнд 18:40:12  356983
>>356971
В таком случае все претензии к Архимеду. Это он придумал, что натуральных чисел бесконечно много. До него люди думали, что чисел лишь конечное количество.
Аноним 18/04/16 Пнд 19:52:18  356995
>>356971
>Пеано дал не определение, а аксиомы которым удовлетворяет множество натуральных чисел.
Задание аксиоматики – один из способов определения объекта в математике.
>Но без самих натуральных чисал заданных каким-то другим способом, ты эти аксиомы не реализуешь, более того не сможешь их даже сформулировать без опоры на индукцию.
Причём здесь индукция и натуральное число?
Аноним 19/04/16 Втр 14:19:06  357146
(33Кб, 1146x228)
(33Кб, 1133x225)
>>356976
Две бурбакицы этому.
Аноним 19/04/16 Втр 14:24:04  357148
(37Кб, 750x600)
>>354754
{,{,{,{,{,{,{,{,{,{,{,{,{}}}}}}}}}}}}}
Аноним 19/04/16 Втр 16:23:00  357166
(24Кб, 632x198)
>>357146
Врать не надо, ок? Вот как Бурбаки на самом деле определяли числа 0 и 1. Тем более в твоей википедии опечатка.
Аноним 19/04/16 Втр 16:38:47  357168
>>357166
Это из первого тома, "теории множеств"? Какая страница? Ок, смотри. 0 и 1 формально определены. Дальнейшее описывается аксиомами Пеано. Вуаля, у нас есть формальное определение N. Оп-хуй соснул.
Аноним 19/04/16 Втр 16:53:45  357171
>>357168
Из первого. Страницы 187 и 188.

Вообще-то чтобы определить натуральные числа, достаточно формально определить ординалы, затем сказать, что конечные ординалы - это ординалы, в точности меньшие первого предельного, и определить натуральные числа как конечные ординалы. Аксиомы Пеано станут на таком пути теоремами.

Проблема в том, что оп не верит в аксиому индуктивного множества в ZFC.
Аноним 20/04/16 Срд 16:26:39  357391
>>357166
Где, в вики, опечатка ? Пруфы завезёшь? Исправим же.
Аноним 22/04/16 Птн 03:53:28  357708
(185Кб, 670x596)
>>357391
Держи примечание советской редакции.
Аноним 22/04/16 Птн 18:38:51  357788
>>357708
Чёт не вижу бурбаков там.
Аноним 22/04/16 Птн 19:43:00  357801
>>295488 (OP)
Просто оставлю это здесь.
https://www.youtube.com/watch?v=vlIA0ujw8lI
Аноним 25/04/16 Пнд 14:51:31  358336
>>357171

Это не вопрос веры, просто если не можешь дать корректное определение нужно в этом расписаться, типа считаем это понятие неопределяемым и интуитивно ясным. Если можешь, то дай корректное определение, без скрытой круговой зависимости.
Аноним 25/04/16 Пнд 15:12:05  358341
>>358336
>без скрытой круговой зависимости.
Конкретнее, что за круговая зависимость?
Аноним 25/04/16 Пнд 15:34:12  358349
>>358341

Давай своё определение N, покажу.
Аноним 25/04/16 Пнд 16:19:15  358364
>>358349
http://www.math.wustl.edu/~kumar/courses/310-2011/Peano.pdf
Аноним 25/04/16 Пнд 17:35:09  358386
>>358341
Это когда ты говоришь определение, а N-петух говорит, что это не определение. И так по кругу.
Аноним 26/04/16 Втр 09:10:54  358503
>>358386

Ну так а что тебя заставляет нести хуйню вместо определения, ммм???
Аноним 26/04/16 Втр 09:12:49  358504
>>358364

Не катит, это во-первых формальная система, которая сама основана на индукции и во-вторых сформулирована в терминах теории множеств, которая мощнее N.
Аноним 26/04/16 Втр 09:41:34  358508
>>358503
Покажи хотя бы одно определение чего угодно.
Аноним 26/04/16 Втр 09:49:51  358511
>>358508

Возьми любое определение не исходного понятия.
Аноним 26/04/16 Втр 10:06:15  358518
>>358511
Приведи пример, пожалуйста.
Аноним 26/04/16 Втр 10:49:52  358523
>>358518

Определение банахова пространства, например.
Аноним 26/04/16 Втр 11:13:57  358529
>>358523
Ну и чем слова "нормированное векторное пространство, полное по метрике, порождённой нормой" отличаются от слов "ординал омега-нуль"?
Аноним 26/04/16 Втр 11:32:14  358532
>>358529

Тем, что в первом случае мы реально выражаем новое понятие через уже определенные, а твой омега нуль, если начать распутывать всю цепочку, в итоге все равно упрётся в индукцию.
Аноним 26/04/16 Втр 13:01:29  358548
>>358532
То есть ты считаешь понятие нормы определённым?
Аноним 26/04/16 Втр 14:39:13  358569
>>358548

Конечно.
Аноним 26/04/16 Втр 21:01:13  358623
>>358504
>основана на индукции
Что это значит?
>мощнее N.
В каком смысле «мощнее»? И почему ты сравниваешь паяльник с арбузом?
Аноним 26/04/16 Втр 21:12:29  358624
Вы уже обсуждали что для определения N не нужна аксиома индукции? N определяют первые 4 аксиомы, а аксиома индукции нужна уже для построения арифметики.
Аноним 26/04/16 Втр 21:37:24  358630
>>358623
Как же вы заебали со своим невежеством и идиотизмом.

другой анон
Аноним 26/04/16 Втр 21:38:53  358631
>>358624
Нет, первые 4 аксиомы без пятой или её аналога N не определяют.
Аноним 27/04/16 Срд 00:21:06  358658
>>358631
Почему нет? Опираясь на первые четыре аксиомы я могу перечислить все натуральные числа - назвать 1 (опираясь на первую аксиому), оттуда получить что число идущее за 1 тоже натуральное (вторая аксиома) и дальше повторять вторую аксиому пока не получу все числа. Опираясь на первые четыре аксиомы я могу определить является ли натуральным произвольное число - сначала проверить не является ли это число 1 (тогда оно натуральное по первой аксиоме) если нет то посмотреть какое число следует перед заданным - и если оно натуральное, то и заданное число натуральное (вторая аксиома). По-моему этого достаточно для определения N. Этого не достаточно для более полного определения арифметики - для многих утверждений нужна аксиома индукции, но N определено.
Аноним 27/04/16 Срд 00:27:09  358659
>>358658
>Опираясь на первые четыре аксиомы я могу перечислить все натуральные числа
Нет, не можешь.

> пока не получу все числа.
Until forever.

> Опираясь на первые четыре аксиомы я могу определить является ли натуральным произвольное число
Нет, не можешь.

> По-моему этого достаточно для определения N.
Ты заблуждаешься.
Аноним 27/04/16 Срд 00:32:43  358661
>>358659
> Нет, не можешь.
Нет, могу.
> Until forever.
Ничего не меняет.
> Нет, не можешь.
Нет, могу. Назви число натуральность которого я не смогу определить опираясь на 4 аксиомы, а буду вынужден применить индукцию.
> Ты заблуждаешься.
Ты этого не показал, а следовательно нет.
Аноним 27/04/16 Срд 00:38:50  358663
>>358661
> Нет, могу.
Я тебе дальше привел аргументы, а ты занимаешься нетты-каньем.

> Ничего не меняет.
Меняет. Ты их никогда не получишь, если ты по-английски не понял.

> Назви число натуральность которого я не смогу определить
-1

> Ты этого не показал, а следовательно нет.
Неверное следование, кстати.

> Ты этого не показал.
Показал. Я рассчитывал на то, что ты сообразительный.
Аноним 27/04/16 Срд 04:03:03  358684
>>358663
Сначала дай определение числу -1, и только потом проси доказать его натуральность. Вдруг под -1 ты понимаешь 1'''''''''?
Аноним 27/04/16 Срд 08:09:02  358690
>>358663
Так как перед числом -1 (так как мы не имеем определения -1) не следует ни какое другое число получаем что перед ним нет натурального числа (аксиома 2) и так как -1 не равно 1 (аксиома 1) то оно не натуральное.
Аноним 27/04/16 Срд 08:13:12  358691
>>358658
>и дальше
Без аксиомы индукции "и дальше" нет.
Аноним 27/04/16 Срд 08:35:38  358693
>>358691
Есть. Конкретно в том случае используется рекурсия. В рекуррентную формулу подставляется 1 как особое задаваемое первой аксиомой значение, после чего рекуррентно подставляется результат формулы в результате чего мы получаем все натуральные числа. Это не имеет ничего общего с индукцией, ну кроме того что рекуррентная формула нужна для использования индукции - для индукции нужно сначала вычислить рекуррентную формулу в каком-нибудь виде.
Аноним 27/04/16 Срд 09:30:59  358695
>>358693

Рекурсия это всего лишь переформулировка индукции только в других терминах, абсолютно ничего не меняет.
Аноним 27/04/16 Срд 09:33:27  358696
>>358658

> и дальше повторять вторую аксиому пока не получу все числа.

Это и есть индукция тащемта. Т.е. ты говоришь индукция - это индукция, ну вот смотрите, показываю на пальцах. Но это не определение, а разъяснение неопределяемого понятия на примерах, точно так же как про множество говорят что это собрание предметов в мешке или прямую как длину без ширины.
Аноним 27/04/16 Срд 13:44:36  358768
>>358696
>Это и есть индукция тащемта
>>358695
>Рекурсия это всего лишь переформулировка индукции
Нет. Рекурсия это всего лишь применение формулы к ее результату. А индукция - аксиоматические утверждение что если существует верное индукционное предположение, и это предположение верно также и для 1 то оно верно для всех натуральных чисел. Общего у индукции и рекурсии нет ничего, кроме того что индукционное предположение сводимо к некоторой рекуррентной формуле.
Если представить индукцию как некоторое множество утверждений то оно будет состоять из подмножества утверждений которые образую рекуррентную формулу и как минимум еще подмножества утверждений относительно числа 1. Так как рекурсия входит как подмножество в множество утверждений определяющих индукцию, при это это множество содержит еще и подмножество определяющее особый случай 1, которое не входит в подмножество утверждений связанных с рекуррентной формулой - то рекурсия не может являться индукцией - индукция содержит большее количество утверждений.
Аноним 27/04/16 Срд 13:51:18  358770
>>358690
>так как мы не имеем определения -1
Кто не имеет? Ну определи его как наименьшее по модулю отрицательное число. Как отрицательные числа из натуральных получить вы надеюсь уже прошли?

>Так как перед числом -1 не следует ни какое другое число
Докажи.

>>358768
> индукция - аксиоматические утверждение
> индукция содержит большее количество утверждений
Господи, какую же ты хуйню несешь. Я надеюсь, что ты троллишь, да?

Еще раз:
> дальше повторять вторую аксиому пока не получу все числа
Ты понимаешь, что ты не можешь "повторять вторую аксиому пока не получу все числа" без аксиомы индукции? Ты понимаешь, что можешь только показать натуральность некоторого конечного количества чисел, но не можешь обобщить этот результат на бесконечное множество, если не используешь пятую аксиому?

Алсо, почитай, что такое рекурсия и что такое индукция, пожалуйста.
https://en.wikipedia.org/wiki/Recursion
https://en.wikipedia.org/wiki/Mathematical_induction
Аноним 27/04/16 Срд 14:08:52  358778
>>358770
> Кто не имеет?
Мы не имеем. А нас в распоряжении аксиомы пеано. Если ты вводишь другие понятия то тебе следует определить их, чтобы было понятно о чем ты говоришь. Если ты соизволил определить -1 как "отрицательное число" то тут и аксиомы не нужны. Отрицательные числа не являются натуральными по определению.
> Докажи
Это прямое следствие из того что -1 не определено. Если определить -1 то (возможно) можно будет говорить про числа которые следуют за ним, в зависимости от определения. Пока -1 не определено - перед ним не может быть никаких чисел.
> хуйню несешь. Я надеюсь, что ты троллишь, да?
Ясно.
> ты не можешь
Могу. В этом смысл рекуррентной формулы.
>Алсо, почитай,
Ты бы сам лучше почитал что там написано. Алсо тебя прямо по предоставленой тобой ссылке обоссали
> Many mathematical axioms are based upon recursive rules. For example, the formal definition of the natural numbers by the Peano axioms can be described as: 0 is a natural number, and each natural number has a successor, which is also a natural number. By this base case and recursive rule, one can generate the set of all natural numbers.
Аноним 27/04/16 Срд 15:25:21  358800
>>358778
>By this base case and recursive rule, one can generate the set of all natural numbers.
>and recursive rule
>recursive rule

Это и есть аксиома индукции, уебок. Зарепортил мудака.
Аноним 27/04/16 Срд 15:38:54  358809
>>358800
Слив засчитан.
Аноним 27/04/16 Срд 15:59:36  358826
>>358800

+1
Аноним 28/04/16 Чтв 09:21:03  359012
>>358809

C точки зрения малолетнего долбоёба - безусловно.
Аноним 29/04/16 Птн 16:40:24  359422
When a mathematician looks a tomato, what does he see? An infinite number of tomato points.
Аноним 03/05/16 Втр 20:41:55  360255
(32Кб, 550x367)
>>359422
Аноним 23/06/16 Чтв 00:59:54  370916
bump
Аноним 24/09/16 Суб 12:41:49  389401
(7Кб, 268x326)
(109Кб, 600x450)
Давайте окончательно расстаивим точки. Натуральное число по Маркову - это слово в алфавите | например. Т.е. | - это 1, ||| - это 3. Производящие схемы для N - это аксиомы Пеано. Даже изначальная интуиция числа как продолжительности отрезка времени по Брауэру имеет доказанную нейрофизиологическую основу - http://www.cell.com/trends/cognitive-sciences/references/S1364-6613(03)00230-4 что дает объяснение почему уже ребенок в детском саду если он не анэнцефал без мозгов типа опа этого треда прекрасно понимает, что такое натуральное число и как с ним работать (пик 2), тогда как формально-аксиоматическое маняопределение единицы у аутистов-бурбаков занимает сколько там квадриллионов знаков и абсолютно лишено какого-либо применения кроме как тралить им дебилов.
Аноним 24/09/16 Суб 13:47:29  389413
>>389401
>дебилов
Ты ведь хотел сказать уважаемых математиков, так!?
Аноним 24/09/16 Суб 13:51:53  389414
>>389413
Уважаемых математиков такой хуетой не затралишь, слишком толсто.
Аноним 24/09/16 Суб 16:36:11  389460
>>295488 (OP)
Что тут определять, ёбтвоюналево?
Натуральные числа = пересечение всех индуктивных множеств со вкинутыми туда операциями и свойствами из теории чисел.

Готово, бля - запилил!
Тред можно закрывать.
Аноним 24/09/16 Суб 18:06:44  389472
>>389401
>интуиционизм
Лол, эта анальная клоунада что вроде кефирщиков из мира физики, или леваков от мира экономики.
Аноним 24/09/16 Суб 18:25:04  389473
>>389472
>анальная клоунада что вроде кефирщиков
Аргументы будут? Так-то даже Брауэр был прав, несмотря на то, что физиологические основы восприятия времени человеком в те годы были неизвестны. Про конструктивизм и говорить нечего, вряд ли даже самый поехавший будет отрицать корректность конструктивного доказательства в математике.
Аноним 07/10/16 Птн 18:08:28  391253
>>389401

Ебаклак, ты определил только три натуральных числа.
Аноним 07/10/16 Птн 18:09:00  391254
>>389460

Анус себе запили, псина.
Аноним 10/10/16 Пнд 13:56:00  391936
(47Кб, 1128x225)
>>391253
Даунятко, я их все определил. 3 из них я только привел в качестве примера.
Аноним 10/10/16 Пнд 15:11:25  391945
>>391936

Ты не определил, а обозвал их палочками и всё. Я с тем же успехом могу тебе дать определение множества предложив называть его хуем.
Аноним 10/10/16 Пнд 15:44:00  391952
>>391945
>Ты не определил, а обозвал их палочками и всё.
Ну раз так сказал школьник с мейлру, то ладно.
Аноним 10/10/16 Пнд 17:23:22  391989
>>391952

C точки зрения малолетнего долбоёба - безусловно.
Аноним 10/10/16 Пнд 17:29:07  391991
(212Кб, 1080x1078)
>>391989
Да-да, мне тут полтора ебалая два треда пытаются рассказать, что конструктивная математика плохая, негодная. И ты так делай. Можешь начать с опровержения MLTT, или хотя бы с алгорифмов Маркова. Более конкретно - расскажи, почему >>391936 не определяет N.
Аноним 10/10/16 Пнд 17:57:13  391996
>>391991

Потому что это масло масляное. Пытаясь определить N, ты просто другими словами приводишь интуитивное описание того же самого.

PS Ты претендуешь на знание конструктивной математики и при этом даже не понял, что в ней конструктивные процессы (суть натуральные числа) это интуитивное, неопределяемое понятие.
Аноним 10/10/16 Пнд 18:05:04  391999
(969Кб, 1388x1738)
>>391996
>при этом даже не понял, что в ней конструктивные процессы (суть натуральные числа) это интуитивное, неопределяемое понятие.
В интуиционизме определяемое через интуицию времени пикрелейтед, в конструктивизме - через задание алфавита и производящих схем. Проблемс?..
Аноним 10/10/16 Пнд 18:32:47  392000
>>391999

И то и другое не является определением, а лишь иллюстрацией на пальцах. Типа как школьникам в геометрии чертят на доске чертежи.

Аноним 10/10/16 Пнд 18:37:03  392002
>>392000
Определение объекта в конструктивизме = задание правил его построения. У нас есть система Поста или что то же самое, алгорифм Маркова для построения натуральных чисел. Проблему ты сам придумал на ровном месте. Ребенок в детском саду прекрасно понимает что такое натуральное число. А ты хочешь какое-то определение, оторванное от самого натурального числа. Т.е. мистику какую-то тебе надо.
10/10/16 Пнд 20:33:25  392008
>>392002
Прекращай, тебя даже твои собратья конструктивисты обосрали в другом треде. Хватит.
Аноним 11/10/16 Втр 09:38:14  392091
>>392008
>тебя
Ты в слове "себя" ошибку сделал. Вопрос остается, какое вообще может быть определение объекта без самого объекта, в отрыве от него.
11/10/16 Втр 15:15:02  392123
>>392091
Хватит позорится. Ты же только себя унижаешь ещё больше.
Аноним 11/10/16 Втр 15:28:44  392124
>>392123
>себя
>себя
>позорится
А ничего, что это Маркова определение? Кроме ad hominem есть и другие аргументы, не, не слышал? Смешно с местных дурачков. В -тся-ться научись для начала, потом про позор рассказывай.
11/10/16 Втр 15:31:49  392125
>>392124
Скажи, Число Грэмма определенно?
Аноним 11/10/16 Втр 15:33:48  392126
>>392125
Я тебе на это давал ответ в соседнем треде, пойди и почитай. Тут речь про N.
11/10/16 Втр 15:35:10  392127
>>392126
Ничего ты не дал. Ты только уходил от ответа. И если ответ есть, прикрекрепи на него сссылку.
Аноним 11/10/16 Втр 16:03:08  392132
>>392127
Нахуй-нахуй, по сто раз одно и то же писать неинтересно. Как и искать конкретный пост. Я отвечал про фактически непостроимые числа, а именно что фактически они и не существуют, раз не построены, есть только правила их построения и т.о. потенциальная построимость. ИТТ же я хочу услышать ОПРАВДАНИЯ почему определение N по Маркову не определяет N.
Аноним 11/10/16 Втр 16:09:27  392134
>>392132

Потому, что оно содержит порочный круг. Точно так же как множество нельзя строго определить как совокупность.
Аноним 11/10/16 Втр 16:12:03  392136
>>392134
>Потому, что оно содержит порочный круг.
Где, какой? Подробнее.
Аноним 11/10/16 Втр 16:22:04  392140
>>392134
>множество нельзя строго определить как совокупность.
Множество как что-то общее, неконкретное, н-р по Кантору, да, нельзя. Рекурсивно-перечислимое, как конкретный конструктивный объект - можно.
Аноним 11/10/16 Втр 16:56:21  392145
>>391991
>Да-да, мне тут полтора ебалая два треда пытаются рассказать, что конструктивная математика плохая, негодная. И ты так делай. Можешь начать с опровержения MLTT, или хотя бы с алгорифмов Маркова
Зачем опровергать какие-то узкие области из Computer Science? Они просто не имеют никакого отношения к математике, по простой причине: их нельзя применять в современном исследовании. Это как нестандартный анализ или алгебра диаграмм: можно на досуге позаниматься, но как дисциплины они абсолютно бесполезны.
Аноним 11/10/16 Втр 16:56:24  392146
>>392136

Использование индукции ("и т.д."). В случае палочек предположение что мы можем к любой совокупности уже написанных палочек приписать еще одну.

>>392140

Ну так марковское определение (да и любое другое) приведенное выше работает на уровне канторовского определения множества.
Аноним 11/10/16 Втр 17:03:16  392148
>>392146
>В случае палочек предположение что мы можем к любой совокупности уже написанных палочек приписать еще одну.
А типа мы не можем? Или что?
>Использование индукции ("и т.д.").
Есть функция следования succ, ее применять тоже нельзя, богородица не велит?
> марковское определение (да и любое другое) приведенное выше работает на уровне канторовского определения множества.
С чего бы? Ты не понимаешь разницы между канторовским множеством и рекурсивно-перечислимым конструктивным?
>>392145
>Зачем опровергать какие-то узкие области из Computer Science? Они просто не имеют никакого отношения к математике, по простой причине: их нельзя применять в современном исследовании.
Раз так сказал эксперт с мейлру, то точно. Конструктивная математика и в частности MLTT не имеет отношение к математике, языки с зависимыми типами нельзя применять, я тебя правильно понял?
Аноним 11/10/16 Втр 17:18:06  392154
>>392148
>Конструктивная математика и в частности MLTT не имеет отношение к математике, языки с зависимыми типами нельзя применять, я тебя правильно понял?
Конечно, нет.

Вот приходит математик на работу и вдруг думает, а попробую-ка я применить конструктивную математику, а почему бы и нет, хуле!
И тут выясняется, что вся основа его предмета ( на деле любой области математики) доказана с помощью индукции на бесконечность, ее надо доказывать заново, причем нетривиальными методами, ведь даже такие утверждения как "простых чисел бесконечно много" предполагают собой поиск рекурсивной последовательности доказанно включающей в себя только простые числа (небось еще открытая проблема, лол).
Ну ладно, думает он, сейчас по-быстренькому докажу. Берет книги уровня undergraduate и начинает доказывать, и тут хуяк. Оказывается надо еще изучать какую-то теорию типов и неклассическую логику. Придется отложить доказательства, а взяться за изучение интуиционистской логики и теории типов.
Наш упорный математик не сдается, ну что уж взялся за гуж, да не говори, что не дюж. Покорно делает все упражнения, читает HoTT утром и Брауэра перед сном. Предположим, он как-то уложил в себя новые знания, а старые индуктивные методы забыл.
Он начинает доказывать все заново и тут вдруг встречает какую-нибудь хитрую логическую схему или абстрактный объект, например, из гомологической алгебры. Он думает: а как мне ее формализовать? И вдруг узнает, что это открытая математическая проблема, да и проблем таких дохуя, даже неясна связь между унивалетностью и параметризацией. В итоге он понимает, что не может продвинуться дальше уровня undergraduate в области, в которой работает всю жизнь, и стреляет себе в голову из двух пистолетов.
Аноним 11/10/16 Втр 17:33:20  392157
>>392148

> А типа мы не можем? Или что?

С на каком основании ты это можешь сделать? Ты как школьник с которого требуют доказательства геометрической теоремы, а он в ответ говорит ну это же очевидно из рисунка, какие тут доказательства нхх.

> Есть функция следования succ, ее применять тоже нельзя, богородица не велит?

Что бы определить эту функцию тебе уже надо опираться на индукцию иначе ты этого не сделаешь чисто технически.

> С чего бы?

C того что оба они относятся к неформальному метаязыку, который дальше не поддается уточнению и формализации без появления замкнутого круга в определениях.

> Ты не понимаешь разницы между канторовским множеством и рекурсивно-перечислимым конструктивным?

В контексте нашего обсуждения разницы нет, так как в обоих случаях используется интуитивное представление множества как совокупности, то что во втором случае это очень ограниченный набор множеств сути не меняет, это лишь попытка отсечь противоречия возникающие в больших множествах типа множества всех множеств.

Аноним 11/10/16 Втр 17:41:56  392158
>>392157
Так а какое определение тебе надо? Ты простую вещь пойми - даже тривиальные определения, оторванные от построимого объекта, и от балды, априорно, обобщенные на все вообще случаи, чреваты противоречиями и парадоксами. Казалось бы, что может быть проще канторовского определения множества? Однако, оно приводит к общеизвестным парадоксам. От этого не застрахованы даже обобщения конструктивных подходов, например парадокс Жирара, к которому приводил первый вариант MLTT.
>>392157
>Что бы определить эту функцию тебе уже надо опираться на индукцию иначе ты этого не сделаешь чисто технически.
Забудь ты про свои индукции, или формалистам про них рассказывай. Функция следования есть конструктивный объект, построимый и т.о. определенный. Где тут индукции?
>C того что оба они относятся к неформальному метаязыку, который дальше не поддается уточнению и формализации без появления замкнутого круга в определениях.
Определение Маркова - это построимый конструктивный объект, не нуждающийся для своего построения ни в каких метаязыках, индукциях и прочих религиозных построениях, не имеющих отношения к самому построимому конструктивному объекту.
>это лишь попытка отсечь противоречия возникающие в больших множествах типа множества всех множеств.
Противоречивыми могут быть только те множества, которые задаются формализмами, оторванными от построимости этих множеств. Конструктивное построимое множество не может быть противоречивым.
Аноним 11/10/16 Втр 18:01:25  392163
>>392158

> Так а какое определение тебе надо? Ты простую вещь пойми - даже тривиальные определения, оторванные от построимого объекта, и от балды, априорно, обобщенные на все вообще случаи, чреваты противоречиями и парадоксами. Казалось бы, что может быть проще канторовского определения множества?

Нужно или дать корректное определение, или признать что его нет и считать это понятие исходным, не определямым. Почему-то с множеством так и поступают, а N почти всегда делают вид что определяют.

> Забудь ты про свои индукции, или формалистам про них рассказывай. Функция следования есть конструктивный объект, построимый и т.о. определенный. Где тут индукции?

Мдэ, тяжелый случай. Ты хоть читал своего Маркова? Он там еще во введении пишет, что считает понятие конструктивного процесса исходным и не определяемым. А оно по своей сути чуть шире интуитивной индукции (конструктивисты любят говорить об абстракции потенциальной осущетвимости, а это суть индукция). А ты мне предлагаешь на основе более широкого неопределямого понятия дать определение N. Ты бы еще из канторовской теории множеств определение N выводил.

> Определение Маркова - это построимый конструктивный объект, не нуждающийся для своего построения ни в каких метаязыках, индукциях и прочих религиозных построениях, не имеющих отношения к самому построимому конструктивному объекту.

Понятие конструктивного процесса у Маркова это метаязыковое неопределяемое интуитивное понятие. Что значит не нуждается, если оно явно или неявно, но содержит это все у себя в пререквизитах?

> Противоречивыми могут быть только те множества, которые задаются формализмами, оторванными от построимости этих множеств. Конструктивное построимое множество не может быть противоречивым.

К чему это высказывание? Речь шла про определение множества, я тебе указал что в обоих случаях они понимаются интуитивно, только в случае конструктивного подхода некоторые множества (которые неконструктивные) запрещены. В моей аналогии с натуарльными числами имелась ввиду только эта связь.

Аноним 11/10/16 Втр 18:09:28  392166
N-шизик и конструктивистопетух нашли друг-друга. Как это мило.
Аноним 11/10/16 Втр 18:10:51  392168
>>392166

Ты кто вообще?
11/10/16 Втр 21:31:10  392191
Сколько петухов надо, чтобы вкрутить лампочку определить N?
Аноним 12/10/16 Срд 00:05:20  392202
>>392132
>Нахуй-нахуй, по сто раз одно и то же писать неинтересно.
Ты написал слово "Брауэер" более 300 раз за последнии три треда. А про слово "конструктивный" я вообще молчу. Давай не будем обманывать друг-друга. Ты только и делаешь что пишешь по сто раз одно и тоже.
>Я отвечал про фактически непостроимые числа, а именно что фактически они и не существуют, раз не построены, есть только правила их построения и т.о. потенциальная построимость.
Ты так и не ответил существуют ли они в виде Да/Нет. И сейчас уходишь от ответа. Ты говорил, что объект существует, если его можно построить. Раз число Грэма нельзя построить, то ты просто берёшь и веруешь в какую-то потенциальную построимость и правила построение. Чем твоя вера в потренциальную посроимость отличаеться от веры в Аллаха? Смотри, если мы будем лететь по космосу, то возможно когда-нибудь найдём Аллаха, то есть Аллах потенциально существует.
Аноним 12/10/16 Срд 23:57:53  392356
Пилите перекот

13/10/16 Чтв 00:44:53  392368
Бамп.
13/10/16 Чтв 00:45:09  392369
https://2ch.hk/sci/res/392366.html
Перекат.
Аноним 13/10/16 Чтв 15:32:56  392407
>>392202
>Ты так и не ответил существуют
Я сто раз ответил - НЕТ, не существует, что именно в слове "нет" тебе не понятно-то?
>Чем твоя вера в потренциальную посроимость отличаеться от веры в Аллаха
Правила построения - сами по себе конструктивный объект, вера в Аллаха и, что одно и то же, актуальную бесконечность - нет.
>Смотри, если мы будем лететь по космосу, то возможно когда-нибудь найдём Аллаха, то есть Аллах потенциально существует.
Я тебе не помню какой уже раз советую - начни с себя, узнай что значит такое слово - "потенциально".
>>392163
Я тебе одно скажу - никакого определения N, оторванного от самих натуральных чисел нет и не может быть в принципе. Конструктивное определение N давным-давно используют в языках с зависимыми типами и прочими пруверами еще с дебрауновского AUT-68. Определение = построение, хотя бы в виде задания правил для такого построения.
Аноним 13/10/16 Чтв 17:00:57  392416
>>392407

> никакого определения N, оторванного от самих натуральных чисел нет и не может быть в принципе.

Распиши это все подробно и откуда ты это взял.
Аноним 13/10/16 Чтв 17:35:26  392419
>>392416
Определение объекта есть правило или правила его построения. Существование объекта есть возможность его построения. Натуральное число есть слово в алфавите |. Палку | ты непосредственно видишь перед собой. Например, ||| это 3. Любые другие "определения" нуждаются в определении определений, определенеии определения определений и т.д. С конструктивным же объектом все просто и понятно, даже если фактически построить его нет возможности из-за нехватки ресурсов, потенциальная возможность построения остается, а сами по себе правила построения есть конструктивный объект, который можно использовать.
14/10/16 Птн 06:11:01  392468
>>392419
>Определение объекта есть правило или правила его построения
С чего это вдруг? Если это не аксиома, то из чего это следует? Нет, правда скажи, из чего ты делаешь такие выводы?
>Существование объекта есть возможность его построения
Ты же сам сказал, что число Грэмма не существует!
>Натуральное число есть слово в алфавите |. Палку | ты непосредственно видишь перед собой. Например, ||| это 3
Ты не определил N, ты ничего не определил, а просто нарисовал три палки. Дай, как угодаю, ты считаешь, что N не надо вообще определять, а они уже как бы вшиты в нашу интуицыю, но итуицыя - последние, чему надо доверять, поскольку она подсказывает, что Земля плоская.
>Любые другие "определения" нуждаются в определении определений, определенеии определения определений и т.д.
Твои тоже. Определение слова, алфавита?
>С конструктивным же объектом все просто и понятно, даже если фактически построить его нет возможности из-за нехватки ресурсов, потенциальная возможность построения остается,
То есть весь конструктивизм основам на вере? Чем твоя возмозможность построения, лучше Аллаха?
>а сами по себе правила построения есть конструктивный объект, который можно использовать
Чем конструктивный объект лучше неконструктивного?
Аноним 14/10/16 Птн 09:36:39  392493
https://en.wikipedia.org/wiki/Surreal_number ещё один охуенный способ определить N ( и не только)
Аноним 14/10/16 Птн 10:36:10  392500
>>392468
>Если это не аксиома, то из чего это следует?
Из фактически построенного конструктивного объекта. Всё бы вам в какие-то там аксиомы веровать.
>Ты не определил N, ты ничего не определил, а просто нарисовал три палки.
Конструктивный объект - это и есть его определение.
>Определение слова, алфавита?
Алфавит состоит из знаков, в данном случае - |, слово - знакосочетание, н-р |||. Конструктивные определения не предполагают верований во что-то отличное от конструктивных объектов, в которые в свою очередь веровать не нужно, т.к. они даны фактически и непосредственно.
Аноним 14/10/16 Птн 11:07:13  392506
ПЕРЕКАТ https://2ch.hk/sci/res/392504.html
14/10/16 Птн 14:54:22  392538
>>392506
Перекатывайтесь в обычный мат-тред https://2ch.hk/sci/res/392366.html
Нечего по всей доске растекаться.

[Назад][Обновить тред][Вверх][Каталог] [Реквест разбана] [Подписаться на тред] [ ] 508 | 28 | 89
Назад Вверх Каталог Обновить

Топ тредов
Избранное